NCLEX Review

Pataasin ang iyong marka sa homework at exams ngayon gamit ang Quizwiz!

2. Please share with me more about your concerns 251 (304)

A 55-year old male client confides in the nurse that he is concerned about his sexual function. What is the nurse's best response? 1. How often do you have a sexual relationship 2. Please share with me more about your concerns 3. You are still young and have nothing to be concerned about 4. You should not have a decline in testosterone until you are in you 80s

3.The flowers from your garden are beautiful, but should not be placed in the child' room at this time

A 6 year old child with leukemia is hospitalized and is receiving a combination chemo. Lab results indicate that the child is neutropenic, and protective isolation procedures are initiated. The grandmother of the child visits and brings a fresh bouquet of flowers picked from her garden and asks the nurse for a vase for the flowers. Which response should the nurse provide to the grandmother? 1. I have a vase in the utility room,and I will get it for you 2. I will get the vase and wash it well before you put the flower in it 3.The flowers from your garden are beautiful, but should not be placed in the child' room at this time 4. When you bring the flowers into the room, place them on the bedside stand as far away from the child as possible

10gtt/min 153 (194)

A HCP prescribes 1 unit of PRBC to infuse over 4 hours. The unit of blood contains 250 mL. The drop factor is 10gtt/1mL. The nurse prepares to set the flow rate at how many drops per minute?

31 drops 146 (193)

A HCP prescribes 1000mL of NS to infuse over 8 hours. The drop factor is 15 drops/ 1mL. The nurse sets the flow rate at how many drops per minute?

125 milliliters 148 (193)

A HCP prescribes 3000mL of D5W to be administered over 24 hours. The nurse determines that how many milliliters per hour will be administered to the client?

16 mL per hour 147 (193)

A HCP prescribes heparin sodium, 1300 units/hr by IV infusion. The pharmacy prepares the medication and delivers an IV bag labeled heparin sodium 20,000 units/250mL. An infusion pump must be used to administer the medication. The nurse sets the infusion pump at how many milliliters per hour to deliver 1300 units/ hour?

8 milliliters 155 (194)

A HCP prescribes regular insulin 8units/hr by IV infusion. The pharmacy prepares the medication and then delivers an IV bag labeled 100 units of regular insulin in 100mL of NS. An infusion pump must be used to administer the medication. The nurse sets the infusion pump at how many milliliters per hour to deliver 8units/hr?

21 drops 141 (193)

A HCP prescription reads 1000mL of NS to infuse over 12 hours. The drop factor is 15 drops/1mL. The nurse prepares to set the flow rate at how many drops per minute?

2 milliliters 144 (193)

A HCP prescription reads clindamycin phosphate 0.3g in 50mL NS to be administered IV over 30 min. The medication label reads clindamycin phosphate 900mg in 6mL. The nurse prepares how many milliliters of the medication to administer the correct dose?

1.5 tablets 150 (193)

A HCP prescription reads levothyroxine, 150mcg orally daily. The medication label reads Synthroid, 0.1mg/tablet. The nurse administers how many tablets to the client?

0.8 milliliters 154 (194)

A HCP prescription reads morphine sulfate, 8mg stat. The medication ampule reads morphine sulfate, 10mg/mL. The nurse prepares how many milliliters to administer the correct dose?

2 capsules 145 (193)

A HCP prescription reads phenytoin 0.2g orally twice daily. The medication label states that each capsule is 100mg. The nurse prepares how many capsules to administer one dose?

15 milliliters 143 (193)

A HCP prescription reads potassium chloride 30mEq to be added to 1000mL NS and to be administered over a 10 hour period. The label on the medication bottle reads 40mEq/20mL. The nurse prepares how many milliliters of potassium chloride to administer the correct dose of medication?

1.3 mL 142 (193)

A HCP prescription reads to administer an IV dose of 400,000 units of penicillin G Benzathine. The label on the 10-mL ampule sent from the pharmacy reads penicillin G benzathine, 3000,000 units/mL. The nurse prepares how much medication to administer the correct dose?

2. Remove all metal-containing objects from the client.

A MRI study is prescribed for a client with a suspected brain tumor. The nurse should implement which action to prepare the client for this test. 1. Shave the groin for insertion of a femoral catheter. 2. Remove all metal-containing objects from the client. 3. Keep the client NPO for 6 hours before the test. 4. Instruct the client in inhalation techniques for the administration of the radioisotope.

3. Page an interpreter from the hospital's interpreter services 232 (294)

A Spanish speaking client arrives at the triage desk in the ED and states to the nurse, "No speak English, need interpreter." Which is the best action for the nurse to take? 1. Have one of the client's Spanish-speaking triage receptionists interpret 2. Have the Spanish speaking triage receptionist interpret 3. Page an interpreter from the hospital's interpreter services 4. Obtain a Spanish-English dictionary and attempt to triage the client

1. Notify the HCP

A child undergoes surgical removal of a brain tumor. During the post-op period,the nurse is monitoring the child and notes that the child is restless, the pulse is elevated, and the BP has decreased significantly from baseline. The nurse suspects the child is in shock. Which is the most appropriate nursing action? 1. Notify the HCP 2. Place the child in a supine position 3. Place the child in Trendelenburg's position 4. Increase the IV fluids

4. Deforoxamine Antidote for too much iron 377 (426)

A child with B-thalassemia is receiving long-term blood transfusion therapy for the treatment of the disorder. Chelation therapy is prescribed as a result of too much iron from the transfusions. Which medication should the nurse anticipate to be prescribed? 1. Fragmin 2. Meropenem 3. Metoprolol 4. Deforoxamine

3. Increased efficiency of contractions Amniotomy is rupture of membranes by HCP 295(355)

A client arrives at a birthing center in active labor. Her membranes are still intact,and the HCP prepares to perform an amniotomy. What will the nurse relay to the client as the most likely outcome of the amniotomy? 1. Less pressure on her cervix 2. Decreased number of contractions 3. Increased efficiency of contractions 4. The need for increased maternal blood pressure monitoring

2. July 26, 2015 257 (310)

A client arrives at the clinic for the first prenatal assessment. She tells the nurse that the first day of her last menstrual period was October 19th, 2014. Using Nagele's rule, which expected date of delivery should the nurse document in the client's chart? 1. July 12, 2014 2. July 26, 2015 3. August 12, 2015 4. August 26, 2015

3. A level that indicates a MI Troponin T <0.2 66 (124)

A client arrives in the ED complaining of chest pain that began 4 hours ago. A troponin T blood specimen is obtained, and the results indicate a level of 0.6. The nurse determines that this result indicates which finding? 1. A normal level 2. A low value that indicates possible gastritis 3. A level that indicates a MI 4. A level that indicates the presence of possible angina

4. Draw a sample for PT and INR PT <9.6-11.8 INR 2-5 73 (124)

A client brought to the ED states that he has accidentally been taking 2 times his prescribed dose of warfarin for the past week. After noting that the client has no evidence of obvious bleeding, the nurse plans to take which action? 1. Prepare to administer an antidote 2. Draw a sample for type and crossmatch and transfuse the client 3. Draw a sample for an activated PTT 4. Draw a sample for PT and INR

1. Slow the IV infusion 103 (159)

A client had a 1000mL bag of 5% dextrose in 0.9% sodium chloride hung at 3pm. The nurse making rounds at 345m; finds that the client is complaining of a pounding headache and is dyspneic, is experiencing chills,and is apprehensive, with an increased pulse rate. The IV bag has 400mL remaining. The nurse should take which action first? 1. Slow the IV infusion 2. Sit the client up in bed 3. Remove the IV catheter 4. Call the HCP

2. 0.9% Sodium chloride 130 (170)

A client has a prescription to recieve a unit of PRBC. The nurse should obtain which IV solution from the IV storage area to hang with the blood product at the client's bedside? 1. LR 2. 0.9% Sodium chloride 3. 5% dextrose in 0.9% NS 4. 5% dextrose in 0.45% NS

2. 15 BUN 8-25 65 (124)

A client has been admitted to the hospital for UTI and dehydration. The nurse determines that the client has received adequate volume replacement if the BUN level drops to which value? 1. 3 2. 15 3. 29 4. 35

2. Temperature and weight 95 (145)

A client has been discharged to home on PN. With each visit, the home care nurse should assess which parameter most closely in monitoring this therapy? 1. Pulse and weight 2. Temperature and weight 3. Pulse and blood pressure 4. Temperature and blood pressure

4. Supine, with the amputated limp supported with pillows 185 (227)

A client has just returned to a nursing unit after an above the knee amputation of the right leg. The nurse should place the client in which position? 1. Prone 2. Reverse Trendelenburg's 3. Supine, with the ambulated limb flat on the bed 4. Supine, with the amputated limp supported with pillows

3. Portable chest-x-ray 114 (160)

A client has just undergone insertion of a central venous catheter at the bedside. The nurse would be sure to check which results before initiating the flow rate of the client's IV solution at 100ml/hr? 1. Serum osmolality 2. Serum electrolytes 3. Portable chest-x-ray 4. I&O

4. Decreased oozing of blood from puncture sites and gums 121 (170)

A client has received a transfusion of platelets. Th ensure evaluates that the client is benefiting most from this therapy if the client exhibits which finding? 1. Increased HCT level 2. Increased HGB level 3. Decline of elevated temperature to normal 4. Decreased oozing of blood from puncture sites and gums

4. Administer oxygen 8-10L by face mask 334 (380)

A client in a postpartum unit complains of sudden sharp chest pain and dyspnea. The nurse notes that the client is tachycardia and the RR is elevated. The nurse suspects a PE. Which should be the initial nursing action? 1. Initiate an IV line 2. Assess the BP 3. Prepare to administer morphine 4. Administer oxygen 8-10L by face mask

2. I will maintain strict bed rest thoughout the remainder of the pregnancy

A client in the first trimester of pregnancy arrives at a clinic and reports that she has been experiencing vaginal bleeding. A threatened abortion is suspected, and the nurse instructs the client regarding management of care. Which statement made by the client indicates a need for further instruction? 1. I will watch for the evidence of the passage of tissue 2. I will maintain strict bed rest thoughout the remainder of the pregnancy 3. I will count the number of perineal pads used on a daily basis and note the amount and color of blood on the pad 4. I will avoid sexual intercourse until the bleeding has stopped, and for 2 weeks following the last evidence of bleeding

1. 5% dextrose in LR 112 (160)

A client involved in a MVC presents to the ED with severe internal bleeding. The client is severely hypotensive and unresponsive. The nurse anticipates that which IV solution will most likely be prescribed to increase intravascular volume, replace immediate blood loss volume, and increase blood pressure? 1. 5% dextrose in LR 2. 1/3 NS 3. 1/4 NS 4. 1/2 NS

2. Lying in bed on the unaffected side 179 (226)

A client is being prepred for a throacentesis. The nurse should assist the client to which position for the procedure? 1. Lying in bed on the affected side 2. Lying in bed on the unaffected side 3. Sim's position with the HOB flat 4. Prone with the head turned to the side and supported by a pillow

2. Decrease PN rate to 50ml/hr 87 (144)

A client is being weaned from PN and is expected to being taking solid food today. The ongoing solution rate has been 100mL/hr. The nurse anticipates that which prescription regarding the PN solution will accompany the diet prescription? 1. Discontinue the PN 2. Decrease PN rate to 50ml/hr 3. Start 0.9% NS at 25ml/hr 4. Continue current infusion rate prescriptions for PN

3. Transport the victim to the operating room for surgery.

A client is brought to the ED by EMS after being hit by a car. The name of the client is unknown, and the client has sustained a severe head injury and multiple fractures and is unconscious. An emergency craniotomy is required. Regarding informed consent for the surgical procedure, which is the best action? 1. Obtain a court order for the procedure. 2. Ask the EMS team to sign the informed consent. 3. Transport the victim to the operating room for surgery. 4. Call the police to identify the client and locate the family.

2. To promote rapid volume expansion 126 (170)

A client is brought to the ED having experienced blood loss related to an arterial laceration. Fresh-frozen plasma is prescribed and transfused to replace fluid and blood loss. The nurse understands that which is the rationale for transfusion fresh-frozen plasma to this client? 1. To treat the loss of platelets 2. To promote rapid volume expansion 3. Because a transfusion must be done slowly 4. Because it will increase the HGB and HCT levels

4. Weakness, thirst, and increased urine output 91 (144)

A client is receiving PN. The nurse monitor the client for complications of the therapy and should assess the client for which manifestations of hyperglycemia? 1. Fever, weak pulse, and thirst 2. N/V and oliguria 3. Sweating, chills,and abdominal pain 4. Weakness, thirst, and increased urine output

4. Leaving the rate of the heparin infusion as is PTT 20-36 sec (doubled when on heparin therapy) 67 (124)

A client is receiving a continuous IV infusion of heparin sodium to treat DVT. The client's activated PTT is 65. The nurse anticipates that which action is needed? 1. Discontinuing the heparin infusion 2. Increasing the rate of the heparin infusion 3. Decreasing the rate of the heparin infusion 4. Leaving the rate of the heparin infusion as is

2. Oranges 85 (136)

A client is recovering from abdominal surgery and has a large abdominal wound. The nurse should encourage the client to eat which food item that is naturally high in vitamin C to promote wound healing? 1. Milk 2. Oranges 3. Bananas 4. Chicken

3. Hypervolemia 102 (145)

A client receiving PN complains of a headache. The nurse notes that the client has an increased blood pressure, bounding pulse, JVD, and crackles bilaterally. The nurse determines that the client is experiencing which complication of PN therapy? 1. Sepsis 2. Air embolism 3. Hypervolemia 4. Hyperglycemia

4. Crackles on auscultation of the lungs 100 (145)

A client receiving PN in the home setting has a weight gain of 5lb in 1 week. The nurse should next assess the client for the presence of which condition? 1. Thirst 2. Polyuria 3. Decreased BP 4. Crackles on auscultation of the lungs

3. Send them to the lab for culture 94(145)

A client receiving PN suddenly develops a fever. The nurse notifies the HCP, and the HCP initially prescribes that the solution and tubing be changed. What should the nurse do with the discontinued materials? 1. Discard them in the unit trash 2. Return them to the hospital pharmacy 3. Send them to the lab for culture 4. Save them for return to the manufacturer

1. Septicemia 118 (169)

A client receiving a transfusion of PRBC begins to vomit. The client's BP is 90/50 from a baseline of 125/78. The client's temperature is 100.8 orally from a baseline of 99.2 orally. The nurse determines that the client may be experiencing which complication of a blood transfusion? 1. Septicemia 2. Hyperkalemia 3. Circulatory overload 4. Delayed transfusion reaction

1,2 128 (170)

A client requiring surgery is anxious about the possible need for a blood transfusion during or after the procedure. The nurse suggests to the client to take which actions to reduce the risk of possible transfusion complications? Select all that apply 1. Ask a family member to donate blood ahead of time 2. Given an autologous blood donation before the surgery 3. Take iron supplements before surgery to boost HGB levels 4. Request that any donated blood be screened twice by the blood bank 5. Take adequate amounts of vitamin C several days prior to the surgery date

1,2,3,5 111 (160)

A client rings the call bell and complains of pain at the site of an IV infusion. The nurse assesses the site and determines that phlebitis has developed. The nurse should take which actions in the care of this client? Select all that apply 1. Notify the HCP 2. Remove the IV catheter at that site 3. Apply warm moist packs to the site 4. Start a new IV in a proximal portion of the same vein 5. Document the occurrence, actions taken, and the client's response

1,2,3,4 175 (217)

A client who has had abdominal surgery complains of feeling as through "something gave way" in the incisional site. The nurse removes the dressing and notes the presence of a loop of bowel protruding through the incision. Which nursing interventions should the nurse take? Select all that apply 1. Contact the surgeon 2. Instruct the client to remain quiet 3. Prepare the client for wound closure 4. Document the finds and actions taken 5. Place a sterile saline dressing and ice packs over the wound 6. Place the client in a supine positing without a pillow under the head

2. HGB 8 Sodium 135-145 HGB 10-12; 12-16 Platelets 150000-400000 creatinine 0.6-1.3 176 (217)

A client who has undergone preadmission testing has had blood drawn for serum lab studies, including a compete blood count, coagulation studies, and electrolytes and creatinine levels. Which lab result should be reported to the surgeon's office by the nurse, knowing that it could cause surgery to be postponed? 1. Sodium 141 2. HGB 8 3. Platelets 210000 4. Serum creatinine 0.8

4. Respiratory acidosis without compensation 58 (112)

A client who is found unresponsive has ABG drawn and the results indicate the following: pH 7.12, PCO2 90, and HCO3 22. The nurse interprets the results as indicating which condition? 1. Metabolic acidosis with compensation 2. Respiratory acidosis with compensation 3. Metabolic acidosis without compensation 4. Respiratory acidosis without compensation

3. Custard 84 (136)

A client who is recovering from surgery has been advanced from a clear liquid diet to a full liquid diet. The client is looking forward to the diet change because he has been "bored" with the clear liquid diet. The nurse should offer which full liquid item to the client? 1. Tea 2. Gelatin 3. Custard 4. Ice pop

4. "The best thing about this is that I can use it anywhere, anytime"

A client with CAD has selected guided imagery to help cope with psychological stress. Which client statement indicates an understanding of this stress reduction measure? 1. "This will help only if I play music at the same time" 2. "This will work for me only if I am alone in a quiet area" 3. "I need to do this only when I lie down in case I fall asleep" 4. "The best thing about this is that I can use it anywhere, anytime"

4. Preventing and recognizing hyperglycemia A1C <6.5 71 (124)

A client with DM has an A1C of 9%. On the basis of this test result, the nurse plans to teach the client about the need for which measure? 1. Avoiding infection 2. Taking in adequate fluids 3. Preventing and recognizing hypoglycemia 4. Preventing and recognizing hyperglycemia

4. Summer squash 81 (136)

A client with HTN has been told to maintain a diet low in sodium. The nurse who is teaching this client about foods that are allowed should include which food item in a list provided to the client? 1. Tomato soup 2. Boiled shrimp 3. Instant oatmeal 4. Summer squash

1. On the left side, with the head lower than the feet 89 (144)

A client with PN infusion has disconnected the tubing from the central line catheter. The nurse assesses the client and suspects an air embolism. The nurse should immediately place the clientin which position? 1. On the left side, with the head lower than the feet 2. On the left side, with the head higher than the feet 3. On the right side, with the head lower than the feet 4. On the right side, with the head higher than the feet

4. Consciously think about walking over imaginary lines on the floor

A client with Parkinson's disease develops akinesia while ambulating, increasing the risk for falls. Which suggestion should the nurse provide to the client to alleviate this problem? 1. Use a wheelchair to move around 2. Stand erect and use a cane to ambulate 3. Keep the feet close together while ambulating and use a walker. 4. Consciously think about walking over imaginary lines on the floor

4. An increased pH with an increased HCO3 55 (111)

A client with a 3-day history of n/v presents to the ED. The client is hypoventilating and has a RR of 10/min. The EKG monitor displays tachycardia, with a HR of 120. ABG are drawn and the nurse reviews the results, expecting to note which finding? 1. a decreased pH and an increased CO2 2. An increased pH and a decreased CO2 3. A decreased pH and a decreased HCO3 4. An increased pH with an increased HCO3

3. Wheezes 240 (294)

A client with a diagnosis of asthma is admitted to the hospital with respiratory distress. Which type of adventitious lung sounds should the nurse expect to hear when performing a respiratory assessment on this client? 1. Stridor 2. Crackles 3. Wheezes 4. Diminished

4. Place the normal report in the client's medical record Platelets 1500-4000 76 (124)

A client with a history of GI bleeding has a platelet count of 300000. The nurse should take which action after seeing the lab results? 1. Report the abnormally low count 2. Report the abnormally high count 3. Place the client on bleeding precautions 4. Place the normal report in the client's medical record

1. 3.2 Potassium 3.5-5 68 (124)

A client with a history of cardiac disease is due for a morning dose of furosemide. Which serum potassium level, if noted in the client's lab report, should be reported before administering the dose of furosemide? 1. 3.2 2. 3.8 3. 4.2 4. 4.8

4. Obtain a telephone consent from a family member, following agency policy 169 (217)

A client with a perforated gastric ulcer is scheduled for surgery. The client cannot sign the operative consent form because of sedation from opioid analgesics that have been administered. The nurse should take which most appropriate action in the care of this client? 1. Obtain a court order for the surgery 2. Have the charge nurse sigh the informed consent immediately 3. Send the client to surgery without the consent for being signed 4. Obtain a telephone consent from a family member, following agency policy

2. Holding the next dose of warfarin PT level 9.6-11.8 63 (124)

A client with a-fib who is receiving maintenance therapy of warfarin sodium (Coumadin) has a PT of 35 sec. On the basis of the PT, the nurse anticipates which prescription? 1. Adding a dose of heparin sodium 2. Holding the next dose of warfarin 3. Increasing the next dose of warfarin 4. Administering the next dose of warfarin

2. Drowsiness

A client with renal insufficiency has a magnesium level of 3.6. On the basis of this lab result, the nurse interprets which sign as significant? 1. Hyperpnea 2. Drowsiness 3. Hypertension 4. Physical hyperactivity

4. Blood-warming device 129 (170)

A client with severe blood loss resulting from multiple trauma requires raid transfusion of several units of blood. The nurse asks another health team member to obtain which device for use during the transfusion procedure to help reduce the risk of cardiac dysrhythmias? 1. Infusion pump 2. Pulse Ox 3. Cardiac monitor 4. Blood-warming device

4. Circulatory overload 115 (160)

A client with the recent diagnosis of MI and impaired renal function is recuperating on the step-down cardiac unit. The client's BP has been borderline low and IV fluids have been infusing at 100ml/hr via a central line catheter in the right internal jugular for approximately 24 hours to increase renal output and maintain the blood pressure. Upon entering the client's room, the nurse notes that the client is breathing rapidly and is coughing. The nurse determines that the client is most likely experiencing which complication of IV therapy? 1. Hematoma 2. Air embolism 3. Systemic infection 4. Circulatory overload

2. Do you plan to have any other children 248 (304)

A couple comes to the family planning clinic and asks about sterilization procedures. Which question by the nurse would determine whether this method of family planning would be most appropriate? 1. Has either of you ever had surgery 2. Do you plan to have any other children 3. Do either of you have DM 4. Do either of you have problems with high BP

4. The presence of Reed-Sternberg cells in the lymph nodes

A diagnosis of Hodgkin's disease is suspected in a 12 year old seen in a clinic. Several diagnostic studies are performed to determine the presence of this disease. Which diagnostic test result will confirm the diagnosis of Hodgkin's disease? 1. Elevated vanillylmandelic acid urinary levels 2. The presence of blast cells in the bone marrow 3. The presence of Epstein-Barr virus in the blood 4. The presence of Reed-Sternberg cells in the lymph nodes

4. "I will call the nursing supervisor to seek assistance regarding your request"

A hospitalized client tells the nurse that a living will is being prepared and that the lawyer will be bringing the will to the hospital today for witness signatures. The client asks the nurse for assistance in obtaining a witness to the will. Which is the most appropriate response to the client? 1. "I will sign as a witness to your signature" 2. "You will need to find a witness on your own" 3. "Whoever is available at the time will sign as a witness for you" 4. "I will call the nursing supervisor to seek assistance regarding your request"

4. Inform the child of bedtime a few minutes before it is time for bed 216 (266)

A mother arrives at a clinic with her toddler and tells the nurse that she has a difficult time getting the child to go to bed at night. What measure is most appropriate for the nurse to suggest to the mother? 1. Allow the child to set bedtime limits 2. Allow the child to have temper tantrums 3. Avoid letting the child nap during the day 4. Inform the child of bedtime a few minutes before it is time for bed

3. Call the Poison Control Center

A mother calls a neighbor who is a nurse and tells the nurse that her 3-year-old child has just ingested liquid furniture polish. The nurse would direct the mother to take which immediate action? 1. Induce vomiting 2. Call an ambulance 3. Call the Poison Control Center 4. Bring the child to the ED

1. A wagon 219 (267)

A mother of a 3 year old asks a clinic nurse about appropriate and safe toys for the child. The nurse should tell the mother that the most appropriate toy for a 3 year old is which? 1. A wagon 2. A golf set 3. A farm set 4. A jack set with marbles

1. Autocratic

A new unit nurse manager is holding her first staff meeting. The manager greets the staff and comments that she has been employed to bring about QI. The manager provides a plan that she developed and a list of tasks and activities for which each staff member must volunteer to perform. In addition, she instructs staff members to report any problem directly to her. What type of leader and manager approach do the new manager's characteristics suggest? 1. Autocratic 2. Situational 3. Democratic 4. Laissez-faire

1. A normal test result 265 (322)

A nonstress test is performed on a client who is pregnant, and the results of the test indicate nonreactive findings. The HCP prescribes on contraction stress test, and the results are documented as negative. How should the nurse document these finding? 1. A normal test result 2. An abnormal test result 3. A high risk for fetal demise 4. The need for a cesarean delivery

3. A client with a fever who is diaphoretic and restless

A nurse has received the assignment for the day shift. After making initial rounds and checking all of the assigned clients, which client should the nurse plan to care for first? 1. A client who is ambulatory 2. A client scheduled for PT at 1 3. A client with a fever who is diaphoretic and restless 4. A post-op client who has just received pain medication

2. Massage the fundus until firm 326 (373)

A nurse is preparing to assess the uterine fundus of a client in the immediate postpartum period. After locating the fundus, the nurse notes that the uterus feels soft and body. Which nursing intervention would be most appropriate? 1. Elevate the client's legs 2. Massage the fundus until firm 3. Ask the client to turn on her left side 4. Push on the uterus to assist in expressing clots

4. A RN leads nursing personnel in providing care to a group of clients

A nursing graduate is attending an agency orientation regarding the nursing model of practice implemented in the health care facility. The nurse is told that the nursing model is a team nursing approach. The nurse understand that planning care delivery will be based on which characteristic of this type of nursing model of practice? 1. A task approach method is used to provide care to clients 2. Managed care concepts and tools are used in providing client care 3. A single RN is responsible for providing care to a group of clients 4. A RN leads nursing personnel in providing care to a group of clients

4. Observing care provided to the client without the client's permission.

A nursing instructor delivers a lecture to nursing students regarding the issue of client's rights and asks a nursing student to identify a situation that represents an example of invasion of client privacy. Which situation, if identified by the student, indicates an understanding of a violation of this client right? 1. Performing a procedure without consent 2. Threatening to give a client a medication 3. Telling the client that he or she cannot leave the hospital. 4. Observing care provided to the client without the client's permission.

3. Connects the umbilical vein to the inferior vena cava 243 (303)

A nursing student is assigned to care for a client in labor. The nursing instructor asks the student to describe fetal circulation, specifically the ductus venosus. Which statement is correct regarding the ductus venosus? 1. Connects the pulmonary artery to the aorta 2. Is an opening between the right and left atria 3. Connects the umbilical vein to the inferior vena cava 4. Connects the umbilical artery to the inferior vena cava

1,4 208 (253)

A parent of a 3-year old tells a clinic nurse that the child is rebelling constantly and having temper tantrums. Using Erikson's psychosocial development theory, which instructions should the nurse provide to the parent? Select all that apply 1. Set limits on the child's behavior 2. Ignore the child when this behavior occurs 3. Allow the behavior, because this is normal at this age period 4. Provide a simple explanation of why the behavior is unacceptable 5. Punish the child ever time the child says no to change the behavior

1,2,3 82 (136)

A post-op client has been placed on a clear liquid diet. The nurse should provide the client with which items that are allowed to be consumed on this diet? Select all that apply 1. Broth 2. Coffee 3. Gelatin 4. Pudding 5. Vegetable juice 6. Pureed vegetables

1. Pneumonia 167 (216)

A post-pp client asks the nurse why it is so important to deep-breathe and cough after surgery. When formulating a response, the nurse incorporates the understanding the retained pulmonary secretions in a post-op client can lead to which condition? 1. Pneumonia 2. Hypoxiemia 3. Fluid imbalance 4. PE

2. Encouraging fluid intake 337 (380)

A postpartum client is diagnosed with cystitis. The nurse should plan for which priority nursing action in the care of the client? 1. Providing sitz baths 2. Encouraging fluid intake 3. Placing ice on the perineum 4. Monitoring HGB and HCT

3. Can you share with me what you've been told about your surgery 170 (217)

A pre-op client expresses anxiety to the nurse about upcoming surgery. Which response by the nurse is most likely to stimulate further discussion between the client and the nurse? 1. If it's any help, everyone is nervous before surgery 2. I will be happy to explain the entire surgical procedure to you 3. Can you share with me what you've been told about your surgery 4. Let me tell you about the care you'll receive after surgery and the amount of pain you can anticipate

4. 14 and 18 259 (310)

A pregnant client asks the nruse in the clinic when she will be able to begin to feel the fetus move. The nurse responds by telling the mother that fetal movements will be noted between which weeks of gestation? 1. 6 and 8 2. 8 and 10 3. 10 and 12 4. 14 and 18

1. Swimming Non-weight baring exercises are allowed 267 (323)

A pregnant client asks the nurse about the types of exercises that are allowable during pregnancy. The nurse should tell that client that which exercise is safest? 1. Swimming 2. Scuba diving 3. Low-impact gymnastics 4. Bicycling with the legs in the air

2. Bend your foot toward your body while extending the knee when the cramps occur 270 (323)

A pregnant client calls a clinic and tells the nurse that she is experiencing leg cramps that awaken her at night. What should the nurse tell the client to provide relief from the leg cramps? 1. Bend your food toward your body while flexing the knee when the cramps occur 2. Bend your foot toward your body while extending the knee when the cramps occur 3. Point your food away from your body while flexing the knee when the cramps occur 4. Point your food away from your body while extending the knee when the cramps occur

2. The vaginal discharge may be bothersome, but is a normal occurrence 263 (322)

A pregnant client in the first trimester calls the nurse at the health care clinic and reports that she has noticed a thin, colorless vaginal drainage. The nurse should make which statement to the client? 1. Come to the clinic immediately 2. The vaginal discharge may be bothersome, but is a normal occurrence 3. Report the the ED at the maternity center immediately 4. Use tampons if the discharge is bothersome, but to be sure to change the tampons every 2 hours

2. Respiration of 10 357 (410)

A pregnant client is receiving mag sulfate for the management of preeclampsia. The nurse determines that the client is experiencing toxicity from the medication if which finding is noted on assessment? 1. Proteinuria of 3 2. Respiration of 10 3. Presence of DTR 4. Serum mag level of 6

3. Inform the client that these contractions are common and may occur throughout the pregnancy 254 (310)

A pregnant client is seen for a regular prenatal visit and tells the nurse that she is experiencing irregular contractions. The nurse determines that she is experiencing Braxton Hicks contractions. On the basis of this finding, which nursing action is most appropriate? 1. Contact the HCP 2. Instruct the client to maintain bed rest of the remainder of the pregnancy 3. Inform the client that these contractions are common and may occur throughout the pregnancy 4. Call the maternity unit and inform them that the client will be admitted in a prelabor condition

4. Isoniazid plus rifampin will be required for 9 months 285 (341)

A pregnant client reports to a clinic, complaining of loss of appetite, weight loss, and fatigue. After assessment of the client, TB is suspected. A sputum culture is obtained and identifies mycobacterium tb. Which instruction should the nurse include in the client's teaching plan? 1. Therapeutic abortion is required 2. She will have to stay at home until treatment is completed 3. Medication will not be started until after delivery of the fetus 4. Isoniazid plus rifampin will be required for 9 months

1. The appearance of the fetal external genitalia 244 (304)

A pregnant client tells the clinic nurse that she wants to know the gender of her baby as soon as it can be determined. The nurse understands that the client should be able to find out the gender at 12 weeks' gestation because of which factor? 1. The appearance of the fetal external genitalia 2. The beginning of differentiation in the fetal groin 3. The fetal testes are descended into the scrotal sac 4. The internal differences in males and females become apparent

3. HGB 9.1 Pica results in iron deficiency anemia 266 (323)

A pregnant client tells the nurse that she has been craving "unusual foods". The nurse gathers additional assessment data and disconvers that the client has been ingesting daily amounts of white clay dirt from her backward. Lab studies are performed and the nurse determines that which finding indicates a physiological consequence of the client's practice? 1. HCT 38% 2. Glucose 86 3. HGB 9.1 4. WBC count 12,400

19.5mL

A prescription reads: acetaminophen liquid, 650mg orally every 4 hours PRN for pain. The medication label reads: 500mg/15ml. The nurse prepares how many milliliters to administer one dose?

2,3,4,5 271 (323)

A rubella titer result of a 1 day postpartum client is less than 1:8, and a rubella virus vaccine is prescribed to be administered before discharge. The nurse provides which information to the client about the vaccine? Select all that apply 1. Breast-feeding needs to be stopped for 3 months 2. Pregnancy needs to be avoided for 1-3 months 3. The vaccine is administered by the subcutaneous route 4. Exposure to immunosuppressed individuals needs to be avoided 5. A hypersensitivity reaction can occur if the client has an allergy to eggs 6. The area of the injection needs to be covered with a sterile gauze for 1 week

4. Apply a thin layer of cream and rub it into the area thoroughly 371 (420)

A topical corticosteroid is prescribed by a HCP for a child with atopic dermatitis (eczema). Which instruction should the nurse give the parent about applying the cream? 1. Apply the cream over the entire body 2. Apply a thick layer of cream to affected areas only 3. Avoid cleansing the area before application of the cream 4. Apply a thin layer of cream and rub it into the area thoroughly

3. 6.4 Protein 6-8 70 (124)

An adult with cirrhosis has been prescribed a diet with optimal amounts of protein. The nurse evaluates the client's status as being most satisfactory if the total protein is which value? 1. 0.4 2. 3.7 3. 6.4 4.9.8

3. Delay hanging the blood and notify the HCP 116 (169)

Packed RBC have been prescribed for a client with low HGB and HCT. The nurse takes the client's temperature before hanging the blood transfusion and records 100.6 orally. Which action should the nurse take? 1. Begin the transfusion as prescribed 2. Administer an antihistamine and begin the transfusion 3. Delay hanging the blood and notify the HCP 4. Administer two tables of acetaminophen and begin he transfusion

4. Place a rolled blanket under the right abdominal flank and hip area 164 (204)

The nursing instructor asks a nursing student to describe the procedure for performing abdominal thrusts on an unconscious pregnant woman at 32 weeks gestation. The student describes a component of the procedure correctly if the student stats that he will take which action? 1. Place his hands on the pelvis to perform the thrusts 2. Perform abdominal thrusts until the object is dislodged 3. Perform left lateral abdominal thrusts until the object is dislodged 4. Place a rolled blanket under the right abdominal flank and hip area

1,2,3,4 247 (304)

The nursing instructor asks a nursing student to list the characteristics of amiotic fluid. The student responds correctly by listing which as characteristics of amniotic fluid? Select all that apply 1. Allows for fetal movement 2. Surrounds, cushions, and protects the fetus 3. Maintains the body temperature of the fetus 4. Can be used to measure fetal kidney function 5. Prevents large particles such as bacteria from passing to the fetus 6. Provides an exchange of nutrients and waste products between the mother and the fetus

3. Two arteries carry deoxygenated blood and waste products away from the fetus to the placenta 242 (303)

The nursing student is preparing to teach a prenatal class about fetal circulation. Which statement should be included in the teaching plan? 1. One artery carries oxygenated blood from the placenta to the fetus 2. Two arteries carry oxygenated blood from the placenta to the fetus 3. Two arteries carry deoxygenated blood and waste products away from the fetus to the placenta 4. Two veins carry blood that is high in carbon dioxide and other waste products away from the fetus to the placenta

2. A child of Mediterranean descent 376 (426)

The nursing student is presenting a clinical conference and discusses the cause of B-thalassemia. The nursing student informs the group that a child at greatest risk of developing this disorder is which one? 1. A child of Mexican decent 2. A child of Mediterranean descent 3. A child whose intake of iron is extremely poor 4. A breast-fed child of a mother with chronic anemia

3. Preparing the medication for bolus administration 39 (99)

The nursing student needs to administer potassium chloride IV as prescribed to a client with hypokalemia. The nursing instructor determines that the student is unprepared for this procedure if the student states that which action is part of the plan for preparation and administration of the potassium? 1. Obtaining and IV infusion pump 2. Monitoring UO during administration 3. Preparing the medication for bolus administration 4. Ensuring that the medication is diluted in the appropriate amount of NS

4. RBC that are microcytic and hypochromic 381(426)

Lab studies are performed for a child suspected to have iron deficiency anemia. The nurse reviews the lab results, knowing that which result indicates this type of anemia? 1. Elevated HGB 2. Decreased reticulocyte count 3. Elevated RBC 4. RBC that are microcytic and hypochromic

4. Peripheral vascular disease 365 (410)

Methylergonovine is prescribed for a client with postpartum hemorrhage. Before administering the med, the nurse contacts the HCP who prescribed the med if which condition is documented in the client's medical history? 1. Hypotension 2. Hypothyroid 3. DM 4. Peripheral vascular disease

2. BP 361 (410)

Methylergonovine is prescribed for a woman to treat postpartum hemorrhage. Before adminsteration of methylergonovine, what is the priority nursing assessment? 1. Uterine tone 2. BP 3. Amount of lochia 4. DTR

3. Massage the fundus until firm 340 (380)

On assessment of a postpartum client,the nurse notes that the uterus feels soft and boggy. The nurse should take which initial action? 1. Elevate the client's legs 2. Document the findings 3. Massage the fundus until firm 4. Push on the uterus to assist in expressing clots

2. Determine whether there are med duplication 230 (275)

The home health nurse is visiting a client for the first time. While assessing the client's medication history, it is noted that there are 19 prescriptions and several over-the-counter medications that the client has been taking. Which intervention should the nurse take first? 1. Check for med interactions 2. Determine whether there are med duplication 3. Call the prescribing HCP and report polypharmacy 4. Determine whether a family member supervises med admin

2,5,6 231 (275)

The long-term care nurse is performing assessments on several of the residents. Which are normal age-related physiological changes the nurse expects to note? Select all that apply 1. Increased HR 2. Decline in visual acuity 3. Decreased RR 4. Decline in long-term memory 5. Increased susceptibility to UTI 6. Increased incidence of awakening after sleep onset

2. Obtain equipment for a manual pelvic exam 307 (364)

The maternity nurse is preparing for the admission of a client in the third trimester of pregnancy who is experiencing vaginal bleeding and has a suspected diagnosis of placenta previa. The nurse reviews the HCP prescriptions and should question which prescription? 1. Prepare the client for an ultrasound 2. Obtain equipment for a manual pelvic exam 3. Prepare to draw a HGB and HCT sample 4. Obtain equipment for external electronic fetal heart rate monitoring

1. Allow the newborn infant to signal a need 202 (252)

The maternity nurse is providing instructions to a new mother regarding the psychosocial development of the newborn infant. Using Erikson's psychosocial development theory, the nurse instructs the mother to take which measures? 1. Allow the newborn infant to signal a need 2. Anticipate all the needs of the newborn infant 3. Attend to the newborn infant immediately when crying 4. Avoid the newborn infant during the first 10 minutes of crying

1. Fine grayish red lines Purple colored lesions indicate systemic conditions Honey-colored crusts are characteristic of impetigo and eczema Clusters of fluid-filled vesicles are seen in herpes 367 (419)

The mother of a 3 year old child arrives at a clinic and tells the nurse that the child has been scratching the skin continuously and has developed a rash. The nurse assesses the child and suspects the presence of scabies. The nurse bases this suspicion on which finding noted on assessment of the child's skin? 1. Fine grayish red lines 2. Purple-colored lesions 3. Thick, honey-colored crusts 4. Clusters of fluid-filled vesicles

1. Palpating the abdomen for a mass

The mother of a 4 year old child tells the pediatric nurse that the child's abdomen seems to be swollen. During further assessment of subjective data, the mother tells the nurse that the child is eating well and that the activity level of the child is unchanged. The nurse, suspects the possibility of Wilms' tumor, should avoid which during physical assessment? 1. Palpating the abdomen for a mass 2. Assessing the urine for the presence of hematuria 3. Monitoring the temp for the presence of fever 4. monitoring the BP for the presence of HTN

1. This is a normal behavior at this age 203 (252)

The mother of a 4-year old child calls the clinic nurse and expresses concern because the child has been masturbating. Using Freud's psychosexual stages of development, the nurse should make which response? 1. This is a normal behavior at this age 2. Children usually begin this behavior at age 8 years 3. The child is very young to begin this behavior and should be brought to the clinic 4. This is not normal behavior, and the child should be seen by the HCP

1. Bring the infant to the clinic 342(398)

The mother of a newborn calls the clinic and reports that when cleaning the umbilical cord, she noticed that the cord was moist and that discharge was present. What is the most appropriate nursing instruction for this mother? 1. Bring the infant to the clinic 2. This is a normal occurrence 3. Increase the number of times that the cord is cleaned per day 4. Monitor the cord for another 24-48 hours and call the clinic if the discharge continues

3. At this age, the child is developing his own personality 206 (253)

The mother of an 8 year old child tells the clinic nurse that she is concerned about the child because the child seems to be more attentive to friends than anything else. Using Erikson's psychosocial development theory, the nurse should make which response? 1. You need to be concerned 2. You need to monitor the child's behavior closely 3. At this age, the child is developing his own personality 4. You need to provide more praise to the child to stop this behavior

4. Prevents an infection called ophthalmia neonatorum from occurring after delivery in a newborn born to a woman with an untreated gonococcal infection 350 (399)

The nurse administers erythromycin ointment to the eyes of a newborn and the mother asks the nurse why this is performed. Which explanation is best for the nurse to provide about neonatal eye prophylaxis? 1. Protects the newborn's eyes from possible infection acquired while hospitalized 2. Prevents cataracts in the newborn born to a woman who is susceptible to rubella 3. Minimizes the spread of microorganisms to the newborn from invasive procedures during labor 4. Prevents an infection called ophthalmia neonatorum from occurring after delivery in a newborn born to a woman with an untreated gonococcal infection

4. PTT 374 (426)

The nurse analyzes the lab results of a child with hemophilia. The nurse understand that which result will most likely be abnormal in this child? 1. Platelet count 2. HCT 3. HGB 4. PTT

1. Initiate bleeding precautions

The nurse analyzes the laboratory values of a child with leukemia who is receiving chemo. The nurse notes that the platelet count is 19,500. On the basis of this result, which intervention should the nurse include in the POC? 1. Initiate bleeding precautions 2. Monitor for signs of infection 3. Monitor temp every 4 hours 4. Initiate protective isolation

4. Identify tasks that can be performed safely in the ICU

The nurse arrives at work and is told to report to the ICU for the day because the ICU is understaffed and needs additional nurses to care for the clients. The nurse has never worked in the ICU. The nurse should take which action first? 1. Call the hospital lawyer. 2. Refuse to float to the ICU 3. Call the nursing supervisor 4. Identify tasks that can be performed safely in the ICU

2. Serous drainage 173 (217)

The nurse assesses a client's surgical incision for signs of infection. Which finding by the nurse would be interpreted as a normal finding at the surgical site? 1. Red, hard skin 2. Serous drainage 3. Purulent drainage 4. Warm, tender skin

2. Document the finding 212 (266)

The nurse assesses the vital signs of a 12 month old infant with a respiratory infection and notes that the respiratory rate is 35 breaths. On the basis of this finding, which action is most important? 1. Administer oxygen 2. Document the finding 3. Notify the HCP 4. Reassess the RR in 15 minutes

3. Drying the infant with a warm blanket Key word EVAPORATION 341 (398)

The nurse assisted with the delivery of a newborn. Which nursing action is most effective in preventing heat loss by evaporation? 1. Warming the crib pad 2. Closing the doors to the room 3. Drying the infant with a warm blanket 4. Turning on the overhead radiant warmer

4. Umbilicus and the xiphoid process 159 (204)

The nurse attempts to relieve an airway obstruction in a 3-year-old conscious child. The nurse performs the abdominal thrust maneuver correctly by standing behind the child, placing her arms under the child's axillae and around the child, and positioning her hands to deliver the thrusts between which areas? 1. Groin and abdomen 2. Umbilicus and the groin 3. Lower abdomen and the chest 4. Umbilicus and the xiphoid process

1. Contact the nursing supervisor

The nurse calls the HCP regarding a new medication prescription because the dosage prescribed is higher than the recommended dosage. The nurse is unable to locate the HCP, and the medication is due to be administered. Which action should the nurse take? 1. Contact the nursing supervisor 2. Administer the dose prescribed 3. Hold the medication until the HCP can be contacted 4. Administer the recommended dose until the HCP can be located

4. Decreased CVP 50 (100)

The nurse caring for a client who has been receiving IV diuretics suspects that the client is experiencing a fluid volume deficit. Which assessment finding would the nurse note in a client with this condition? 1. Long congestion 2. Decreased hematocrit 3. Increased blood pressure 4. Decreased CVP

1. Metabolic acidosis 56 (111)

The nurse caring for a client with an ileostomy understands that the client is most at risk for developing which acid-base disorder? 1. Metabolic acidosis 2. Metabolic alkalosis 3. Respiratory acidosis 4. Respiratory alkalosis

3. Prolonged QT interval 43 (99)

The nurse caring for a client with hypocalcemia would expect to note which change on the EKG? 1. Widened T wave 2. Prominent U wave 3. Prolonged QT interval 4. Shortened ST segment

1. Do nothing, because this is an expected finding 199 (243)

The nurse caring for a client with pneumothorax and who has had a chest tube inserted notes continuous gentle bubbling in the suction control chamber. What action is most appropriate? 1. Do nothing, because this is an expected finding 2. Check for an air leak because the bubbling should be intermittent 3. Increase the suction pressure so that the bubbling becomes vigorous 4. Immediately clamp the chest tube and notify the HCP

4. The client who has sustained a traumatic burn 52 (100)

The nurse caring for a group of clients reviews the lab results and notes a potassium level of 5.5 on one client's lab report. The nurse understand that which client is most at risk for the development of a potassium value at this level? 1. The client with colitis 2. The client with Cushing's syndrome 3. The client who has been overusing laxatives 4. The client who has sustained a traumatic burn

1. Hold the feeding 198 (243)

The nurse checks for residual before administering a bolus tube feeding to a client with a NG tube and obtains a residual amount of 150 mL. What is the most appropriate action for the nurse to take? 1. Hold the feeding 2. Reinstill the amount and continue with administering the feeding 3. Elevate the HOB to 45 and administer the feeding 4. Discard the residual amount and proceed with administering the feeding

2. Notify the HCP Digoxin level 0.5-2 64 (124)

The nurse checks the lab result for a serum digoxin level that was prescribed for a client earlier in the day and notes that the result is 2.4. The nurse should take which immediate action? 1. Check the client's last pulse rate 2. Notify the HCP 3. Record the normal value on the client's flow sheet 4. Administer the next dose of medication as scheduled

3. Run NS at a keep-vein-open rate 119 (169)

The nurse determines that a client is having a transfusion reaction. After the nurse stops the transfusion, which action should be taken next? 1. Remove the IV line 2. Run a solution of 5% dextrose in water 3. Run NS at a keep-vein-open rate 4. Obtain a culture of the tip of the catheter device removed from the client

2. Maintaining standard precautions at all times while caring for the newborn 352 (399)

The nurse develops a POC for a woman with HIV and her newborn. The nurse should include which intervention in the POC? 1. Monitoring the newborn's VS routinely 2. Maintaining standard precautions at all times while caring for the newborn 3. Initiating referral to evaluate blindness, deafness, learning problems, or behavioral problems 4. Instructing the breast-feeding mother regarding the treatment of the nipples with nystatin ointment

3. Bed rest with elevation of the affected extremity Want to decrease venous pressure 181 (227)

The nurse develops a plan of care for a client with DVT. Which client position or activity in the plan should be included? 1. Out-of-bed activities as desired 2. Bed rest with affected extremity kept flat 3. Bed rest with elevation of the affected extremity 4. Best rest with the affected extremity in a dependent position

2,3,4,6 207 (253)

The nurse educator is preparing to conduct a teaching session for the nursing staff regarding the theories of growth and development and plans to discuss Kohlberg's theory of moral development. What information should the nurse include in the session? Select all that apply 1. Individuals move through all six stages in a sequential fashion 2. Moral development progresses in relationship to cognitive development 3. A person's ability to make moral judgments develops over a period of time 4. The theory provides a framework for understanding how individuals determine a moral code to guide their behavior 5. In stage 1 (punishment-obedience orientation), children are expected to reason as mature members of society 6. In stage 2 (instrumental-relativist orientation), the child conforms to rules to obtain rewards or have favors returned

1. "It is a process of learning a different culture to adopt to a new or changing environment"

The nurse educator is providing in-service education to the nursing staff regarding transcultural nursing care; a staff member asks the nurse educator to describe the concept of acculturation. The nurse educator should make which most appropriate response? 1. "It is a process of learning a different culture to adopt to a new or changing environment" 2. "It is a subjective perspective of the person's heritage and a sense of belonging to a group" 3. "It is a group of individuals in a society who are culturally distinct and have a unique identity" 4. "It is a group that shares some of the characteristics of the larger population group of which is a part?

3. Call the nursing supervisor and report the incident

The nurse employed in a hospital is waiting to receive a report from the lab via the fax machine. The fax machine activates and the nurse expect the report, but instead receives a sexually oriented photograph. Which is the most appropriate nursing action? 1. Call the police 2. Cut up the photo and throw it away 3. Call the nursing supervisor and report the incident 4. Call the lab and ask for the individual's name who sent the photo

4. A client requiring abdominal wound irrigation and dressing changes every 3 hours

The nurse employed in a long-term care facility is planning assignments for the clients on a nursing unit. The nurse needs to assign four clients and has a LPN, and three UAP's on a nursing team. Which client would the nurse most appropriately assign to the LPN? 1. A client who requires a bed bath 2. An older client requiring frequent ambulation 3. A client who requires hourly vital sign measurements 4. A client requiring abdominal wound irrigation and dressing changes every 3 hours

4. A client with chest pain who states that he just ate pizza that was made with a very spicy sauce

The nurse employed in an emergency department is assigned to triage clients coming to the emergency department for treatment on the evening shift. The nurse should assign priority to which client? 1. A client complaining of muscle aches, a headache, and malaise 2. A client who twisted her ankle when she fell while rollerblading 3. A client with a minor laceration on the index finger sustained while cutting an eggplant 4. A client with chest pain who states that he just ate pizza that was made with a very spicy sauce

3. Activate the fire alarm RACE 137 (184)

The nurse enters a client's room and finds that the wastebasket is on firs. The nurse immediately assists the client out of the room. What is the next nursing action? 1. Call for help 2. Extinguish the fire 3. Activate the fire alarm 4. Confine the fire by closing the room door

4. The mother washes and dries her hands before and after self-care of the perineum and asks for a pair of gloves before feeding 289 (341)

The nurse evaluates the ability of a hepatitis b positive mother to provide safe bottle-feeding to her newborn during postpartum hospitalization. Which maternal action best exemplifies the mother's knowledge of potential disease transmission to the newborn? 1. The mother requests the window be closed before feeding 2. The mother holds the newborn properly during feeding and burping 3. The mother tests the temperature of the formula before initiating feeding 4. The mother washes and dries her hands before and after self-care of the perineum and asks for a pair of gloves before feeding

1. Rotate the bag gently 104 (159)

The nurse has a prescription to hang a 1000ml IV bag of 5% dextrose in water with 20mEq of potassium chloride and needs to add the medication to the IV bag. The nurse should plan to take which action immediately after injecting the potassium chloride into the port of the IV bag? 1. Rotate the bag gently 2. Attach the tubing to the client 3. Prime the tubing with the IV solution 4. Check the solution for yellowing discoloration

2. Rest between contractions 304 (356)

The nurse has been working with a laboring client and notes that she has been pushing effectively for 1 hour. What is the client's primary physiological need at this time? 1. Ambulation 2. Rest between contractions 3. Change positions frequently 4. Consume oral food and fluids

3. I need to continue to take aspirin until the day of surgery 172 (217)

The nurse has conducted pre-op teaching for a client scheduled for surgery in 1 week. The client has a history of arthritis and has been taking acetylsalicylic acid. The nurse determines that the client needs additional teaching if the client makes which statement? 1. Aspirin can cause bleeding after surgery 2. Aspirin can cause my ability to clot blood to be abnormal 3. I need to continue to take aspirin until the day of surgery 4. I need to check with my HCP about the need to stop the aspirin before the scheduled suregery

2. Monitoring the FHR Dystocia is labor that is prolonged or harder than expected 315 (365)

The nurse has developed a POC for a client experiencing dystocia and includes several nursing actions in the POC. What is the priority nursing action? 1. Providing comfort measures 2. Monitoring the FHR 3. Changing the client's position frequently 4. Keeping the significant other informed of the progress of the labor

4. I should wear knee-high hose, but i should not leave them on longer than 8 hours 269 (323)

The nurse has instructed a pregnant client in measures to prevent varicose veins during pregnancy. Which statement by the client indicates a need for further instruction? 1. I should wear panty hose 2. I should wear support hose 3. I should wear flat nonslip shoes that have good support 4. I should wear knee-high hose, but i should not leave them on longer than 8 hours

1. Reassess the client.

The nurse has just assisted a client back to bed after a fall. The nurse and HCP have assess the client and have determined that the client is not injured. After completing the incident report, the nurse should implement which action next? 1. Reassess the client. 2. Conduct a staff meeting to describe the fall. 3. Document in the nurse's notes that an incident report was completed. 4. Contact the nursing supervisor to update information regarding the fall.

1. UO of 20mL/hr 166 (216)

The nurse has just reassessed the condition of a post-op client who was admitted 1 hour ago to the surgical unit. The nurse plans to monitor which parameter most carefully during this next hour? 1. UO of 20mL/hr 2. Temperature of 99.6 3. BP of 100/70 4. Serous drainage on the surgical dression

2. 15 minutes 123 (170)

The nurse has just received a prescription to transfuse a unit of PRBC for an assigned client. Approximately how long will the nurse need to stay with the client to ensure that a transfusion reaction is not occurring? 1. 5 minutes 2. 15 minutes 3. 30 minutes 4. 45 minutes

3. An in-line filter 120 (169)

The nurse has just received a unit of PRBC from the blood bank for transfusion to an assigned client. The nurse is careful to select tubing especially made for blood products, knowing that this tubing is manufactured with which item? 1. An air vent 2. Tinted tubing 3. An in-line filter 4. A microdrip chamber

4. Drawing one line through the error, initialing and dating, and the documenting the correct information.

The nurse has make an error in a narrative documentation of an assessment finding on a client and obtains the client's record to correct the error. The nurse should take which action to correct the error? 1. Documenting a late entry into the client's record. 2. Trying to erase the error for space to write in the correct data. 3. Using whiteout to delete the error to write in the correct data. 4. Drawing one line through the error, initialing and dating, and the documenting the correct information.

1. Vital signs 122 (170)

The nurse has obtained a unit of blood from the blood bank and has checked the blood bag properly with another nurse. Just before beginning the transfusion, the nurse should assess which priority item? 1. Vital signs 2. Skin color 3. Urine output 4. Latest HCT level

1. Normal In a nonstress test, the baseline FHR must be between 120-160. Two or more accelerations of at least 15bpm must occur each lasting at least 15 seconds in a 20 minute interval 264 (322)

The nurse has performed a nonstress test on a pregnant client and is reviewing the fetal monitor strip. The nurse interprets the tests at reactive. How should the nurse document this finding? 1. Normal 2. Abnormal 3. The need for further evaluation 4. That findings were difficult to interpret

1. I will begin abdominal exercises immediately 298 (355)

The nurse has provided discharge instructions to a client who delivered a healthy newborn by cesarean delivery. Which statement made by the client indicates a need for further teaching? 1. I will begin abdominal exercises immediately 2. I will notify the HCP if i develop a fever 3. I will turn on my side and push up with my arms to get out of bed 4. I will lift nothing heavier than my newborn baby for at least 2 weeks

1. Have you ever had a transfusion before 117 (169)

The nurse has received a prescription to transfuse a client with a unit of packed RBC. Before explaining the procedure to the client, the nurse should ask which initial question? 1. Have you ever had a transfusion before 2. Why do you think you need the transfusion 3. Have you ever gone into shock for any reason in the past 4. Do you know the complications and risks of a transfusion

4. The client who has a BS level of 50mg and complains of blurred vision

The nurse has received the client assignment for the day. Which client should the nurse assess first? 1. The client who has a NG tube attached to intermittent suction 2. The client who needs to receive SQ insulin before breakfast 3. The client who is 2 days post-op and is complaining of incisional pain 4. The client who has a BS level of 50mg and complains of blurred vision

1. "I should use a straight razor to shave under my arms."

The nurse has reinforced discharge instructions to a client who has undergone a right mastectomy with axillary lymph node dissection. Which statement by the client indicates a need for further instruction regarding home care measures? 1. "I should use a straight razor to shave under my arms." 2. "I need to be sure that I do not have blood pressures or blood drawn from my right arm." 3. "I should inform all of my other HCP that I have had this surgery." 4. "I need to be sure to wear thick mitt hand covers or use thick pot holders when I am cooking and touching hot pans."

3. The client was found lying on the floor.

The nurse hears a client calling our for help, hurries down the hallway to the client's room, and find the client lying on the floor. The nurse performs an assessment, assists the client back to bed, notifies the HCP of the incident, and completes an incident report. Which statement should the nurse document on the incident report? 1. The client fell out of bed. 2. The client climbed over the side rail. 3. The client was found lying on the floor. 4. The client became restless and tried to get out of bed.

2,3,4,5

The nurse identifies low-risk therapies to a client and should include which therapy(s) in the discussion? Select all that apply 1. Herbs 2. Prayer 3. Touch 4. Massage 5. Relaxation 6. Acupuncture

3. I should avoid exercise because of the negative effects on insulin production 280 (340)

The nurse implements a teaching plan for a pregnant client who is newly diagnosed with gestational diabetes. Which statement made by the client indicates a need for further teaching? 1. I should stay on the diabetic diet 2. I should perform glucose monitoring at home 3. I should avoid exercise because of the negative effects on insulin production 4. I should be aware of any infections and report signs of infection immediately to my HCP

2. I need to lie flat on my back to perform the procedure The patient should be side-lying 272 (323)

The nurse in a clinic is instructing a pregnant client how to perform "kick counts." Which statement by the client indicates a need for further instruction? 1. I will record the number of movements or kicks 2. I need to lie flat on my back to perform the procedure 3. If i count fewer than 10 kicks in 2 hours i should count the kids again over the next 2 hours 4. I should place my hands on the largest part of my abdomen and concentrate on the fetal movements to count the kicks

4. Persistent nonreassuring FHR 312 (365)

The nurse in a labor room is monitoring a client with dysfunctional labor for signs of fetal or maternal compromise. Which assessment finding would alert the nurse to a compromise? 1. Maternal fatigue 2. Coordinated uterine contractions 3. Progressive changes in the cervix 4. Persistent nonreassuring FHR

2. Place the client in Trendelenburg's position 319 (365)

The nurse in a labor room is performing a vaginal assessment on a pregnant client in labor. The nurse notes the presence of the umbilical cord protruding from the vagina. What is the first nursing action with this finding? 1. Gently push the cord into the vagina 2. Place the client in Trendelenburg's position 3. Find the closest telephone and page the HCP 4. Call the delivery room to notify the staff that the client will be transported immediately

1. Provide pain relief measures 313 (365)

The nurse in a labor room is preparing to care for a client with hypertonic uterine contractions. The nurse is told that the client is experiencing uncoordinated contractions that are erratic in their frequency, duration, and intensity. What is the priority nursing action? 1. Provide pain relief measures 2. Prepare the client for an amniotomy 3. Promote ambulation every 30 minutes 4. Monitor the oxytocin infusion closely

1. We want to attend a support group 288 (341)

The nurse in a maternity unit is providing emotional support to a client and her husband who are preparing to be discharged from the hospital after the birth of a dead fetus. Which statement made by the client indicates a component of the normal grieving process? 1. We want to attend a support group 2. We never want to try to have a baby again 3. We are going to try to adopt a child immediately 4. We are okay, and we are going to try to have another baby immediately

3. A gravida II who has just been diagnosed with dead fetus syndrome 282 (340)

The nurse in a maternity unit is reviewing the clients' records. Which client would the nurse identify as being the most risk for developing disseminated intravascular coagulation? 1. A primigravida with mild preeclampsia 2. A primigravida who delivered a 10 pound infant 3 hours ago 3. A gravida II who has just been diagnosed with dead fetus syndrome 4. A gravida IV who delivered 8 hours ago and has lost 500mL of blood

1. Tachypnea and retractions Acrocyanosis is linked with inadequate perfusion 345 (398)

The nurse in a newborn nursery is monitoring a preterm newborn for respiratory distress syndrome. Which assessment finding would alert the nurse to the possibility of this syndrome? 1. Tachypnea and retractions 2. Acrocyanosis and grunting 3. Hypotension and bradycardia 4. Presence of a barrel chest and acrocyanosis

2. Connect the resuscitation bag to the oxygen outlet 343 (398)

The nurse in the NICU receives a call to prepare for the admission of a 43 week gestation newborn with Apgar scores of 1 and 4. In planning for admission of this newborn, what is the nurse's highest priority? 1. Turn on the apnea and cardiorespiratory monitors 2. Connect the resuscitation bag to the oxygen outlet 3. Set up the IV line with 5% dextrose in water 4. Set the radiant warmer control temp at 97.6

1. Administer oxygen via face mask 292 (354)

The nurse in the labor room is caring for a client in the active stage of the first phase of labor. The nurse is assessing the fetal patterns and notes a late deceleration on the monitor strip. What is the most appropriate nursing action? 1. Administer oxygen via face mask 2. Place the mother in a supine position 3. Increase the rate of the oxytocin intravenous infusion 4. Document the finding and continue to monitor the fetal patterns

2. Hemorrhage 317 (365)

The nurse in the postpartum unit is caring for a client who has just delivered a newborn infant following a pregnancy with a placenta previa. The nurse reviews the POC and prepares to monitor the client for which risk associated with placenta previa? 1. Infection 2. Hemorrhage 3. Chronic HTN 4. DIC

1. Cream of wheat, blueberries, coffee 79 (136)

The nurse instructs a client with CKD who is receiving hemodialysis about dietary modifications. The nurse determines that the client understand these dietary modifications if the client selects which items from the dietary menu? 1. Cream of wheat, blueberries, coffee 2. Sausage and eggs, banana, orange juice 3. Bacon, cantaloupe melon, tomato juice 4. Cured pork, grits, strawberries, orange juice

1. Left Sims' position 184 (227)

The nurse is administering a cleansing enema to a client with a fecal impaction. Before administering the enema, the nurse should place the client in which position? 1. Left Sims' position 2. Right Sim's position 3. On the left side of the body, with the HOB elevated to 45 4. On the right side of the body, with the HOB elevated to 45

2. Assess the baseline fetal heart rate 301 (355)

The nurse is admitting a pregnant client to the labor room and attaches an external electronic fetal monitor to the client's abdomen. After attachment of the electronic fetal monitor, what is the next nursing action? 1. Identify the types of accelerations 2. Assess the baseline fetal heart rate 3. Determine the intensity of the contractions 4. Determine the frequency of the contractions

3. The client passively flexes the hip and knee in response to neck flexion and reports pain in the vertebral column 239 (294)

The nurse is assessing a client for meningeal irritation and elicits a positive Brudzinski's sign. Which finding did the nurse observe? 1. The client rigidly extends the arms with pronated forearms and plantar flexion of the feet 2. The client flexes a leg at the hip and knee and reports pain in the vertebral column when the leg is extended 3. The client passively flexes the hip and knee in response to neck flexion and reports pain in the vertebral column 4. The client's upper arms are flexed and held tightly to the sides of the body and the legs are extended and internally rotated

3. Notify the HCP 335 (380)

The nurse is assessing a client in the fourth stage of labor and notes that the fundus if firm, but that bleeding is excessive. Which should the initial nursing action? 1. Record the findings 2. Massage the fundus 3. Notify the HCP 4. Place the client in Trendelenburg's position

3.Instruct the client to request help when getting out of bed 312 (373)

The nurse is assessing a client who is 6 hours postpartum after delivering a full-term health newborn. The client complains to the nurse of feelings of faintness and dizziness. Which nursing action would be most appropriate? 1. Raise the HOB 2. Obtain HGB and HCT 3.Instruct the client to request help when getting out of bed 4. Inform the nursery room nurse to avoid bringing the newborn to the client until the mother's symptoms have subsided

1. Twitching 42 (99)

The nurse is assessing a client with a suspected diagnosis of hypocalcemia. Which clinical manifestation would the nurse expect to note in the client? 1. Twitching 2. Hypoactive bowel sounds 3. Negative Trousseau's sign 4. Hypoactive deep tendon reflexes

3. Document the findings 344 (398)

The nurse is assessing a newborn after circumcision and notes that the circumcised area is red with a small amount of bloody drainage. Which nursing action is most appropriate? 1. Apply gentle pressure 2. Reinforce the dressing 3. Document the findings 4. Contact the HCP

3. Constant crying 347 (398)

The nurse is assessing a newborn who was born to a mother who is addicted to drugs. Which assessment finding would the nurse expect to note during the assessment of this newborn? 1. Lethargy 2. Sleepiness 3. Constant crying 4. Cuddles when being held

2. Uterine tenderness 306 (364)

The nurse is assessing a pregnant client in the second trimester of pregnancy who was admitted to the maternity unit with a suspected diagnosis of abruptio placentae. Which assessment finding should the nurse expect to note if this condition is present? 1. Soft abdomen 2. Uterine tenderness 3. Absence of abdominal pain 4. Painless, bright red vaginal bleeding

1. I will need to increase my insulin dosage during the first 3 months of pregnancy 284 (341)

The nurse is assessing a pregnant client with tpe 1 DM about her understanding regarding changing insulin needs during pregnancy. The nurse determines that further teaching is needed if the client makes which statement? 1. I will need to increase my insulin dosage during the first 3 months of pregnancy 2. My insulin dose will likely need to be increased during my second and third trimesters 3. Episodes of hypoglycemia are more likely to occur during the first 3 months of pregnancy 4. My insulin needs should return to normal within 7 to 10 days after birth if i am bottle-feeding

1. 3.5 191 (242)

The nurse is assessing for correct placement of a NG tube. The nurse aspirates the stomach contents and checks the gastric pH. The nurse verifies correct tube placement if which pH value is noted? 1. 3.5 2. 7.0 3. 7.35 4. 7.5

3,4,5,6 194 (242)

The nurse is assessing the functioning of a chest tube drainage system in a client who has just returned from the recovery room following a thoracotomy with wedge resection. Which are the expected assessment findings? Select all that apply 1. Excessive bubbling in the water seal chamber 2. Vigorous bubbling in the suction control chamber 3. Drainage system maintained below the client's chest 4. 50mL of drainage in the drainage collection chamber 5. Occlusive dressing in place over the chest tube insertion site 6. Fluctuation of water in the tube in the water seal chamber during inhalation and exhalation

3. 300 Amylase 25-151 74 (124)

The nurse is assigned to a 40-year-old client who has a diagnosis of chronic pancreatitis. The nurse anticipates the client's serum amylase level to be which value? 1.45 2. 100 3. 300 4.500

2. The client with kidney disease 51 (100)

The nurse is assigned to care for a group of clients. On review of the clients' medical records, the nurse determines that which client is at risk for fluid volume excess? 1. The client taking diuretics 2. The client with kidney disease 3. The client with an ileostomy 4. The client who requires GI suctioning

1. A client with an ileostomy 49 (99)

The nurse is assigned to care for a group of clients. On review of the clients' medical records, the nurse determines that which client is most likely at risk for a fluid volume deficit? 1. A client with an ileostomy 2. A client with HF 3. A client on long-term corticosteroid therapy 4. A client receiving frequent wound irrigations

4. A client receiving nasal oxygen who had difficulty breathing during the previous shift

The nurse is assigned to care for four clients. In planning client rounds, which client should the nurse assess first? 1. A client scheduled for a chest x-ray 2. A client requiring daily dressing changes 3. A post-op client preparing for discharge 4. A client receiving nasal oxygen who had difficulty breathing during the previous shift

4. Perform the valsava maneuver 195 (243)

The nurse is assisting a HCP with the removal of a chest tube. The nurse should instruct the client to take which action? 1. Exhale slowly 2. Stay very still 3. Inhale and exhale quickly 4. Perform the valsava meneuver

2. Discontinue the infusion of oxytocin 305 (356)

The nurse is assisting a client undergoing induction of labor at 41 weeks' gestation. The client's contractions are moderate and occuring every 2-3 minutes with a duration of 60 seconds. An internal fetal HR monitor is in pace. The baseline FHR has been 120-122bpm for the past hour. What is the priority nursing action? 1. Notify the HCP 2. Discontinue the infusion of oxytocin 3. Place oxygen on at 8-10L 4. Contact the client's primary support person if not currently present

1,2,3,4 253 (310)

The nurse is assisting in performing an assessment on a client who suspects that she is pregnant and is checking the client for probably signs of pregnancy. Which are probably signs of pregnancy? Select all that apply 1. Ballottement 2. Chadwick's sign 3. Uterine enlargement 4. Braxton Hicks contractions 5. Fetal heart rate detected by a nonelectronic device 6. Outline of fetus via radiography or ultrasound

4. The client with lochia that is red and has a foul smelling odor 327 (373)

The nurse is caring for 4 1 day postpartum clients. Which client would require further nursing action? 1. The client with mild afterpains 2. The client with a pulse of 60 3. The client with colostrum discharge from both breasts 4. The client with lochia that is red and has a foul smelling odor

2,3,6 373 (420)

The nurse is caring for a child who sustained a burn injury plans care based on which pediatric considerations? Select all that apply 1. Scarring is less severe in a child than in an adult 2. A delay in growth may occur after a burn injury 3. An immature immune system presents an increased risk of infection for infants and young children 4. The lower proportion of body fluid to mass in a child increases the risk of cardiovascular problems 5. Fluid resuscitation is unnecessary unless burned area is more than 25% of the TBSA 6. Infants and young children are at increased risk for protein and calorie deficiency because they are smaller muscle mass and less body fat than adults

1. Stridor 197 (243)

The nurse is caring for a client immediately after removal of the ET tube. The nurse should report which sign immediately if experienced by the client? 1. Stridor 2. Occasional pink-tinged sputum 3. Respiratory rate of 24 4. A few basilar lung crackles on the right

4. Document the findings and tell the mother that the pattern on the monitor indicates fetal well-being 300 (355)

The nurse is caring for a client in labor and is monitoring the FHR patterns. The nurse notes the presence of episodic accelerations on the electronic fetal monitor tracing. Which action is most appropriate? 1. Notify the HCP of the findings 2. Reposition the mother and check the monitor for changes in the fetal tracing 3. Take the mother's VS and tell the mother that bed rest is required to conserve oxygen 4. Document the findings and tell the mother that the pattern on the monitor indicates fetal well-being

3. The cervix is dilated completely 291 (354)

The nurse is caring for a client in labor. Which assessment finding indicates to the nurse that the client is beginning the second stage of labor? 1. The contractions are regular 2. The membranes have ruptured 3. The cervix is dilated completely 4. The client begins to expel clear vaginal fluid

2. On the nonoperative side with the legs abducted 182 (27)

The nurse is caring for a client who is 1 day post-op for a total hip replacement. Which is the best position in which the nurse should place the client? 1. Slide-lying on operative side 2. On the nonoperative side with the legs abducted 3. Side-lying with the affected leg internally rotated 4. Side-lying with the affected leg externally rotated

2. Potassium level of 3.0 61(112)

The nurse is caring for a client who is on a mechanical ventilator. Blood gas results indicate a pH of 7.50 and a PCO2 of 30. The nurse has determined that the client is experiencing respiratory alkalosis. Which lab value would most likely be noted in this condition? 1. Sodium level of 145 2. Potassium level of 3.0 3. Magnesium level of 2.0 4. Phosphorus level of 4.0

2. Increased pulse rate

The nurse is caring for a client who just returned from the recovery room after undergoing abdominal surgery. The nurse should monitor for which early sign of hypovolemic shock? 1. Sleepiness 2. Increased pulse rate 3. Increased depth of respirations 4. Increase orientation to surroundings

3. Hyperactive bowel sounds 46 (99)

The nurse is caring for a client with HF who is receiving high doses of a diuretic. On assessment, the nurse notes that the client has flat neck veins, generalized muscle weakness, and diminished deep tendon reflexes. The nurse suspects hyponatremia. What additional signs would the nurse expect to note in a client with hyponatremia? 1. Muscle twitches 2. Decreased UO 3. Hyperactive bowel sounds 4. Increased specific gravity for urine

3. An increase in blood pressure 36 (98)

The nurse is caring for a client with HF. On assessment, the nurse notes that the client is dyspneic and crackles are audible on auscultation. What additional signs would the nurse expect to note in this client if excess fluid volume is present? 1. Weight loss 2. Flat neck and hand veins 3. An increase in blood pressure 4. Decreased CVP

2. Metabolic alkalosis 54 (111)

The nurse is caring for a client with a NG tube that is attached to low suction. The nurse monitors the client, knowing that the client is at risk for which acid-base imbalance? 1. Metabolic acidosis 2. Metabolic alkalosis 3. Respiratory acidosis 4. Respiratory alkalosis

2. Place the tube in a bottle of sterile water 192 (242)

The nurse is caring for a client with a chest tube turns the client to the side and the chest tube accidentally disconnects from the water seal chamber. Which initial nursing action should the nurse take? 1. Call the HCP 2. Place the tube in a bottle of sterile water 3. Immediately replace the chest tube system 4. Place a sterile dressing over the disconnection site

1. 2000 WBC 4500-11000 72(124)

The nurse is caring for a client with a diagnosis of cancer who is immunosuppressed. The nurse would consider implementing neutropenic precautions if the client's WBC count was which value? 1. 2000 2. 5800 3. 8400 4.11500

2. Elevate and immobilize the grafted extremity 186 (227)

The nurse is caring for a client with a severe burn who is scheduled for an autograft to be placed on the lower extremity. The nurse develops a post-op plan of care for the client and should include which intervention in the plan? 1. Maintain the client in a prone position 2. Elevate and immobilize the grafted extremity 3. Maintain the grafted extremity in a flat position 4. Keep the grafted extremity covered with a blanket

1. Pork 86 (136)

The nurse is caring for a client with cirrhosis of the liver. To minimize the effects of the disorder, the nurse teaches the client about foods that are high in thiamine. The nurse determines that the client has the best understanding of the dietary measures to follow if the client states an intention to increase the intake of which food? 1. Pork 2. Milk 3. Chicken 4. Broccoli

4. Respirations that are abnormally deep, regular, and increased in rate 57 (112)

The nurse is caring for a client with diabetic ketoacidosis and documents that the client is experiencing Kussmaul's respirations. On the basis of this documentation, which pattern did the nurse observe? 1. Respirations that cease for several seconds 2. Respirations that are regular but abnormally slow 3. Respirations that are labored and increased in depth and rate 4. Respirations that are abnormally deep, regular, and increased in rate

1. Private room or cohort client 140 (184)

The nurse is caring for a client with meningitis and implements which transmission-based precautions for this client? 1. Private room or cohort client 2. Personal respiratory protection device 3. Private room with negative airflow pressure 4. Mask worn by staff when the client needs to leave the room

2. A 42-year-old client who has had an open cholecystectomy 96 (144)

The nurse is caring for a group of adult clients on an acute care medical-surgical nursing unit. The nurse understands that which client would be the least likely candidate for PN? 1. A 66-yr-old client with extensive burns 2. A 42-year-old client who has had an open cholecystectomy 3. A 27-year-old client with severe exacerbation of Crohn's disease 4. A 35-year-old client with persistent nausea and vomiting from chemo

1. The client who is taking diuretics Clients with hyperaldosteronism, Cushing's and taking corticosteroids are at an increased risk for hypernatremia 45 (99)

The nurse is caring for a group of clients reviews the lab results and notes a sodium level of 130 o one client's lab report. The nurse understand that which client is at highest risk for the development of a sodium value at this level? 1. The client who is taking diuretics 2. The client with hyperaldosteronism 3. The client with Cushing's syndrome 4. The client who is taking corticosteroids

2. Elevates the HOB

The nurse is caring for a hospitalized client with a diagnosis of heart failure who suddenly complains of SOB and dyspnea. The nurse should take which immediate action? 1. Administers oxygen to the client 2. Elevates the HOB 3. Calles the HCP 4. Prepares to administer furosemide (Lasix)

3. Secure all connections in the PN system 101 (145)

The nurse is caring for a restless client who is beginning nutritional therapy with PN. The nurse should plan to ensure that which action is taken to prevent the client from sustaining injury? 1. Calculate daily I&O 2. Monitor the temperature once daily 3. Secure all connections in the PN system 4. Monitor blood glucose levels every 12 hours

4. Allowing the client to choose social activities 223 (274)

The nurse is caring for an older client in a long term care facility. Which action contributes to encouraging autonomy in the client? 1. Planning meals 2. Decorating the room 3. Scheduling hair appointments 4. Allowing the client to choose social activities

1. Client's temperature 92 (145)

The nurse is changing the central line dressing of a client receiving PN and notes that the catheter insertion site appears reddened. The nurse should next ass which item? 1. Client's temperature 2. Expiration date on the bag 3. Time of last dressing change 4. Tightness of tubing connections

3. Large stacking blocks

The nurse is choosing age-appropriate toys for a toddler. Which toy is the best choice for this age? 1. Puzzle 2. Toy soldier 3. Large stacking blocks 4. A card game with large pictures

2. G2 T1 P0 A0 L1 261 (311)

The nurse is collecting data during an admission assessment of a client who is pregnant with twins. The client has a healthy 5 year old child who was delivered at 38 weeks and tells the nurse that she does not have a history of any type of abortion or fetal demise. Using GTPAL, what should the nurse document in the client's chart? 1. G3, T2 P0 A0 L1 2. G2 T1 P0 A0 L1 3. G1 T1 P1 A0 L1 4. G2 T0 P0 A0 L1

375 mL 105 (159)

The nurse is completing a time tape for a 1000ml IV bag that is scheduled to infuse over 8 hours. The nurse has just placed the 11am marking at the 500ml level. The nurse would place the mark for noon at which numerical level on the time tape? Fill in the blank

2. Vitamin B12 Vitamin B12 is found in animal products 80 (136)

The nurse is conducting a dietary assessment on a client who is on a vegan diet. The nurse provides dietary teaching and should focus on foods high in which vitamin that may be lacking in a vegan diet? 1. Vitamin A 2. Vitamin B12 3. Vitamin C 4. Vitamin E

3. It promotes the fertilized ovum's normal implantation in the top portion of the uterus 246 (304)

The nurse is conducting a prenatal class on the female reproductive system. When a client in the class asks why the fertilized ovum stays in the fallopian tube for 3 days, what is the nurse's best response? 1. It promotes the fertilized ovum's chances of survival 2. It promotes the fertilized ovum's exposure to estrogen and progesterone 3. It promotes the fertilized ovum's normal implantation in the top portion of the uterus 4. It promotes the fertilized ovum's exposure to lutenizing hormone and follicle-stimulating hormone

1,2,3,4,6 383 (427)

The nurse is conducting a staff in-service training on von Willebrand's disease. Which should the nurse include as characteristics of von Willebrand's disease? Select all that apply 1. Easy bruising 2. Gum bleeding 3. It is a hereditary bleeding disorder 4. Treatment and care are similar to that for hemophilia 5. It is characterized by extremely high creatinine levels 6. The disorder causes platelets to adhere to damaged endothelium

4. The best results are achieved when sitting up or with the head of the bed elevated 45 to 90 degrees 171 (217)

The nurse is conducting pro-op teaching with a client about the use of an incentive spirometer. The nurse should include which piece of information in discussions with the client? 1. Inhale as rapidly as possible 2. Keep a loose seal between the lips and the mouthpiece 3. After maximum inspiration, hold the breath for 15 seconds and exhale 4. The best results are achieved when sitting up or with the head of the bed elevated 45 to 90 degrees

1. Increase in pulse rate Increase in about 10-15bpm 275 (324)

The nurse is describing cardiovascular system changes that occur during pregnancy to a client and understand which findings would be normal for a client i the second trimester? 1. Increase in pulse rate 2. Increase in BP 3. Frequent bowel elimination 4. Decrease in RBC production

3. Prepare an ice pack for application to the area 339 (380)

The nurse is developing a POC for a postpartum client with a small vulvar hematoma. The nurse should include which specific action during the first 12 hours after delivery? 1. Assess VS every 4 hours 2. Measure fundal height every 4 hours 3. Prepare an ice pack for application to the area 4. Inform the HCP of assessment findings

3. Have the client void immediately before going into surgery 168 (217)

The nurse is developing a plan of care for a client scheduled for surgery.. The nurse should include which activity in the nursing care plan for the client on the day of surgery? 1. Avoid oral hygiene and rinsing with mouthwash 2. Verify that the client has not eaten for the last 24 hours 3. Have the client void immediately before going into surgery 4. Report immediately any slight increase in BP or pulse

2. Uses a cup to drink 214 (266)

The nurse is evaluating the developmental level of a 2 year old. Which does the nurse expect to observe in this child? 1. Uses a fork to eat 2. Uses a cup to drink 3. Pours own milk into cup 4. Uses a knife for cutting food

4. Cover the client, raise the side rails, tell the client that you will return shortly, and administer the pain med to the other client

The nurse is giving a bed bath to an assigned client when a UAP enters the client's room and tells the nurse that another assigned client is in pain and needs pain medication. Which is the most appropriate nursing action? 1. Finish the bed bath and then administer the pain medication to the other client 2. Ask the UAP to find out when the last pain med was given to the client 3. Ask the UAP to tell the client in pain that med will be administered as soon as the bed bath is complete 4. Cover the client, raise the side rails, tell the client that you will return shortly, and administer the pain med to the other client

4. Every 30 minutes 134 (184)

The nurse is giving a report to an UAP who will be caring for a client who has had restrains. The nurse instructs the UAP to check the skin integrity of the restrained hands how frequently? 1. Every 2 hours 2. Every 3 hours 3. Every 4 hours 4. Every 30 minutes

4. Pull back on the tube and wait until the respiratory distress subsides 200 (243)

The nurse is inserting a NG tube in an adult client. During the procedure,the client begins to cough and has difficulty breathing. What is the most appropriate nursing action? 1. Quickly insert the tube 2. Notify the HCP immediately 3. Remove the tube and reinsert when the respiratory distress subsides 4. Pull back on the tube and wait until the respiratory distress subsides

4. Blood return shows in the backflash chambers of the catheter 107 (160)

The nurse is inserting an IV line into a client's vein. After the initial stick, the nurse would continue to advance the catheter in which situation? 1. The catheter advances easily 2. The vein is distended under the needle 3. The client does not complain of discomfort 4. Blood return shows in the backflash chambers of the catheter

3. Smoked sausage 83 (136)

The nurse is instructing a client with HTN on the importance of choosing foods low in sodium. The nurse should teach the client to limit intake of which food? 1. Apples 2. Bananas 3. Smoked sausage 4. Steamed vegetables

2. Administer the iron through a straw 380 (426)

The nurse is instructing the parents of a child with iron deficiency anemia regarding the administration of a liquid oral iron supplement. Which instruction should the nurse tell the parents? 1. Administer the iron at mealtimes 2. Administer the iron through a straw 3. Mix the iron with cereal to administer 4. Add the iron formula for easy administration

1. After a shower or bath 238 (294)

The nurse is intructing a client how to perform a testicular self-exam. The nurse should explain that which is the best time to perform this exam? 1. After a shower or bath 2. While standing to void 3. After having a bowel movement 4. While laying in bed before arising

2. 10% dextrose in water You want the highest amount of glucose you can infuse 98 (145)

The nurse is making initial rounds at the beginning of the shift and notes that the PN bag of an assigned client is empty. Which solution readily available on the nursing unit should the nurse hand until another PN solution is mixed and delivered to the nursing unit? 1. 5% dextrose in water 2. 10% dextrose in water 3. 5% dextrose in LR 4. 5% dextrose in NS

3. Infiltration 160 (160)

The nurse is making initial rounds onthe nursing unit to assess the condition of assigned clients. The nurse notes that a client's IV site is cool, pale, and swollen, and the solution is not infusion. The nurse concludes that which complication has occurred? 1. Infection 2. Phlebitis 3. Infiltration 4. Thrombosis

2. Document the finding 213 (266)

The nurse is monitoring a 3 month old infant for signs of increased intracranial pressure. On palpation of the fontanels, the nurse notes that the anterior fontanel is soft and flat. On the basis of this finding, which nursing action is most appropriate? 1. Increase oral fluids 2. Document the finding 3. Notify the HCP 4. Elevate the HOB to 90

1. Vomiting

The nurse is monitoring a 3 year old child for signs and symptoms of ICP after a crainotomy. The nurse plans to monitor for which early s/s of ICP? 1. Vomiting 2. Bulging anterior fontanel 3. Increasing head circumference 4. Complaints of a frontal headache

2. Notify the HCP 384 (436

The nurse is monitoring a child for bleeding after surgery for removal of a brain tumor. The nurse checks the head dressing for the presence of blood and notes a colorless drainage on the back of the dressing. Which intervention should the nurse perform immediately? 1. Reinforce the dressing 2. Notify the HCP 3. Document the finding and continue to monitor 4. Circle the area of drainage and continue to monitor

2. Neurological assessment 366 (419)

The nurse is monitoring a child with burns during treatment for burn shock. The nurse understands that which assessment provides the most accurate guide to determine the adequacy of fluid resuscitation? 1. Skin turgor 2. Neurological assessment 3. Level of edema at burn sight 4. Quality of peripheral pulses

1. Notify the HCP 299 (355)

The nurse is monitoring a client in active labor and notes that the client is having contractions every 3 minutes that last 45 seconds. The nurse notes that the FHR between contractions is 100bpm. Which nursing actions is most appropriate 1. Notify the HCP 2. Continue monitoring the FHR 3. Encourage the client to continue pushing with each contraction 4. Instruct the client's coach to continue to encourage breathing techniques

4. Variable deceleration 296 (355)

The nurse is monitoring a client in labor. The nurse suspects umbilical cord compression if which is noted on the external monitor tracing during a contraction? 1. Variability 2. Accelerations 3. Early deceleration 4. Variable deceleration

1,4,5 358 (410)

The nurse is monitoring a client in preterm labor who is receiving IV mag sulfate. The nurse should monitor for which adverse effects of this medication? 1. Flushing 2. HTN 3. Increased UO 4. Depressed RR 5. Extreme muscle weakness 6. Hyperactive DTR

2. An increase in the pulse from 88-102 330 (379)

The nurse is monitoring a client int he immediate postpartum period for signs of hemorrhage. Which sign, if noted, would be an early sign of excessive blood loss? 1. Temperature of 100.4 2. An increase in the pulse from 88-102 3. A BP change from 130/88-124/80 4. An increase in RR from 18-22

1. Hypotonic 310 (364)

The nurse is monitoring a client who is in the active stage of labor. The client has been experiencing contractions that are short, irregular, and weak, the nurse documents that the client is experiencing which type of dystocia? 1. Hypotonic 2. Precipitous 3. Hypertonic 4. Preterm labor

3. Uterine hyperstimulation 356 (410)

The nurse is monitoring a client who is receiving oxytocin to induce labor. Which assessment finding could cause the nurse to immediately discontinue the oxytocin infusion? 1. Fatigue 2. Drowsiness 3. Uterine hyperstimulation 4. Early deceleration of the FHR

1. Changes in VS 338 (380)

The nurse is monitoring a postpartum client who received epidural anesthesia for delivery for the presence of a vulvar hematoma. Which assessment finding would best indicate the presence of a hematoma? 1. Changes in VS 2. Signs of heavy bruising 3. Complains of intense pain 4. Complains of a tearing sensation

3. Heavy 329 (373)

The nurse is monitoring the amount of lochia drainage in a client who is 2 hours postpartum and notes that the client has saturated a perineal pad in 1 hour. How should the nurse document this finding? 1. Scant 2. Light 3. Heavy 4. Excessive

3. Ensure that the fat emulsion infusion rate is infusion at the prescribed rate 99 (145)

The nurse is monitoring the status of a client's fat emulsion infusion and notes that the infusion is 1 hour behind. Which action should the nurse take? 1. Adjust the infusion rate to catch up over the next hour 2. Increase the infusion rate to catch up over the next 2 hours 3. Ensure that the fat emulsion infusion rate is infusion at the prescribed rate 4. Adjust the infusion rate to run wide open until the solution is back on time

1. Increasing restlessness 174 (217)

The nurse is monitoring the status of a post-op client. The nurse would become most concerned with which sign that could indicate an evolving complication? 1. Increasing restlessness 2. A pulse of 86 3. BP of 110/70 4. Hypoactive bowel sounds in all four quadrants

3. 100 times per minute 161 (204)

The nurse is performing CPR on an infant. When performing chest compressions, the nurse compresses at least how many times? 1. 60 times per minute 2. 80 times per minute 3. 100 times per minute 4. 160 times per minute

3. A significant sway when the client stands erect with feet together, arms at the side, and the eyes closed 233 (294)

The nurse is performing a neurological assessment on a client and elicits a positive Romberg's sign. The nurse makes this determination based on which observation? 1. An involuntary rhythmic, rapid, twitching of the eyeballs 2. A dorsiflexion of the ankle and great toe with fanning of the other toes 3. A significant sway when the client stands erect with feet together, arms at the side, and the eyes closed 4. A lack of normal sense of position when the client is unable to return extended fingers to a point of reference

2. FHR of 180 293 (355)

The nurse is performing an assessment of a client who is scheduled for a cesarean delivery. Which assessment finding would indicate the need to contact the HCP? 1. HGB of 11 2. FHR of 180 3. Maternal pulse rate of 85 4. WBC of 12000

2. 30 cm 252 (310)

The nurse is performing an assessment of a pregnant client who is at 28 weeks of gestation. The nurse measures the fundal height in centimeters and expects which finding? 1. 22 cm 2. 30 cm 3. 36 cm 4. 40 cm

3. FHR of 180 260 (311)

The nurse is performing an assessment of a primigravida who is being evaluated in a clinic during her second trimester of pregnancy. Which finding concerns the nurse and indicated the need for follow up? 1. Quickening 2. Braxton Hicks contractions 3. FHR of 180 4. Consistent increase in fundal height

1,5

The nurse is performing an assessment on a 10 year old child suspected to have Hodgkin's disease. The nurse understands that which assessment finding are specifically characteristic of this disease? Select all that apply 1. Abdominal pain 2. Fever and malaise 3. Anorexia and weight loss 4. Painful, enlarged inguinal lymph nodes 5. Painless, firm, and moveable adenopathy in the cervical area

4,5,6 318 (365)

The nurse is performing an assessment on a client diagnosed with placenta previa. Which of these assessment finding would the nurse expect to note? Select all that apply 1. Uterine rigidity 2.Uterine tenderness 3. Severe abdominal pain 4. Bright red vaginal bleeding 5. Soft, relaxed, nontender uterus 6. Fundal height may be greater than expected for gestational age

3. Notify the HCP 1st trimester 160-170 Near term 110-160 245 (304)

The nurse is performing an assessment on a client who is at 38 weeks' gestation and notes that the fetal heart rate is 174. On the basis of this finding, what is the priority nursing action? 1. Document the finding 2. Check the mothers HR 3. Notify the HCP 4. Tell the client that the FHR is normal

4. Evidence of bleeding, such as in the gums, petechiae, and purpura 281 (340)

The nurse is performing an assessment on a pregnant client with a diagnosis of severe preeclampsia. The nurse reviews the assessment finding and determines that which finding is most closely associated with a complication of this diagnosis? 1. Enlargement of the breasts 2. Complains of feeling hot when the room is cool 3. Periods of fetal movement followed by quiet periods 4. Evidence of bleeding, such as in the gums, petechiae, and purpura

1. Crusting 229 (274)

The nurse is performing an assessment on an older adult client. Which assessment data would indicate a potential complication associated with the skin? 1. Crusting 2. Wrinkling 3. Deepening of expression lines 4. Thinning and loss of elasticity in the skin

3. I drink hot chocolate before bedtime 227 (274)

The nurse is performing an assessment on an older client who is having difficulty sleeping at night. Which statement by the client indicates the need for further teaching regarding measures to improve sleep? 1. I swim three times a week 2. I have stopped smoking cigars 3. I drink hot chocolate before bedtime 4. I read for 40 minutes before bedtime

2. The client has a history of cardiac disease 309 (364)

The nurse is performing an initial assessment on a client who has just been told that a pregnancy test is positive. Which assessment finding indicates that the client is at risk for preterm labor? 1. The client is 35 primigravida 2. The client has a history of cardiac disease 3. The client's HGB level is 13.5 4. The client is 20 primigravida of average weight and height

1. 6-8 160 (204)

The nurse is performing rescue breathing on a 7-year-old client. The nurse delivers one breath per how many seconds to the child? 1. 6-8 2. 8-10 3. 10-12 4. 12-14

2. Maintaining safety because of low blood glucose levels 353 (399)

The nurse is planning care for a newborn of a mother with DM. What is the priority nursing consideration for this newborn? 1. Developmental delays because of excessive size 2. Maintaining safety because of low blood glucose levels 3. Chocking because of impaired suck and swallow reflexes 4. Elevated body temp because of excess fat and glycogen

1. Client pain level 323 (373)

The nurse is planning care for a postpartum client who had a vaginal delivery 2 hours ago. The client had a midline episiotomy and has several hemorrhoids. What is the priority nursing consideration for this client? 1. Client pain level 2. Inadequate UO 3. Client perception of body changes 4. Potential for imbalanced body fluid volume

3. Cream cheese 78 (136)

The nurse is planning to teach a client with malabsorption syndrome about the necessity of following a low-fat diet. The nurse develops a list of high-fat foods to avoid and should include which food item on the list? 1. Oranges 2. Broccoli 3. Cream cheese 4. Broiled haddock

4. Monitor the newborn's response to feedings and weight gain patterns 349 (399)

The nurse is preparing a POC for a newborn with fetal alcohol syndrome. The nurse should include which priority intervention in the POC? 1. Allow the newborn to establish own sleep-rest pattern 2. Maintain the newborn in a brightly lighted area of the nursery 3. Encourage frequent handling of the newborn by staff and parents 4. Monitor the newborn's response to feedings and weight gain patterns

2. Obtain new IV tubing 109 (160)

The nurse is preparing a continuous IV infusion at the medication cart. As the nurse goes to insert the spike end of the IV tubing into the IV bag, the tubing drops and the spike end hits the tip of the medication cart. The nurse should take which action? 1. Obtain a new IV bag 2. Obtain new IV tubing 3. Wipe the spike end of the tubing with betadine 4. Scrub the spike end of the tubing with an alcohol swab

1,2,3,4 331 (380)

The nurse is preparing a list of self-care instructions for a postpartum client who was diagnosed with mastitis. Which instructions should be included on the list? Select all that apply 1. Wear a supportive bra 2. Rest during the acute phase 3. Maintain a fluid intake of at least 3000 4. Continue to breast-feed if the breast are not too sore 5. Take the prescribed antibiotics until the soreness subsides 6. Avoid decompression of the breasts by breast-feeding or breast pump

4. The administration of blood and blood products is not allowed.

The nurse is preparing a plan of care for a client who is a Jehovah's Witness. The client has been told that surgery is necessary. The nurse considers the client's religious preferences in developing the plan of care and should document which information? 1. The client believes the soul lives on after death. 2. Medication administration is not allowed. 3. Surgery is prohibited in this religious group. 4. The administration of blood and blood products is not allowed.

2. Intratracheal 362 (410)

The nurse is preparing to administer beractant to a premature infant who has respiratory distress syndrome. The nurse plans to adminster the mediation by which route? 1. Intradermal 2. Intratracheal 3. Subcutaneous 4. Intramuscular

3. Clamp the NG tube for 30-60 min following admin of the med 190 (242)

The nurse is preparing to administer medication through a NG tube that is connected to suction. To administer the medication, the nurse should take which action? 1. Position the client supine to assist in med absorption 2. Aspirate the NG tube after med admin to maintain patency 3. Clamp the NG tube for 30-60 min following admin of the med 4. Change the suction setting to low intermittent suction for 30 min after med admin

4. Crayons and a coloring book 218 (267)

The nurse is preparing to care for a 5 year old who has been placed in traction following a fracture of the femur. The nurse plans care, knowing that which is the most appropriate activity for this child? 1. A radio 2. A sports video 3. Large picture books 4. Crayons and a coloring book

2. Required NG suction 37 (98)

The nurse is preparing to care for a client with a potassium deficit. The nurse reviews the client's record and determines that the client was at risk for developing the potassium deficit because of which situation? 1. Sustained tissue damage 2. Required NG suction 3. Has a history of Addison's Disease 4. Is taking a potassium-retaining diuretic

3,5,6 221(267)

The nurse is preparing to care for a dying client, and sever family members are at the client's bedside. Which therapeutic techniques should the nurse use when communicating with the family? Select all that apply 1. Discourage reminiscing 2. Make the decision for the family 3. Encourage expressing of feelings, concerns, and fears 4. Explain everything that is happening to all family members 5. Touch and hold the client's or family member's hand if appropriate 6. Be hones and let the client and family know that they will not be abandoned by the nurse

4,5,6 351 (399)

The nurse is preparing to care for a newborn receiving phototherapy. Which interventions should be included in the POC? Select all that apply 1. Avoid stimulation 2. Decrease fluid intake 3. Expose all of the newborn's skin 4. Monitor the skin temp closely 5. Reposition the newborn every 2 hours 6. Cover the newborn's eyes with eye shields or patches

4. Bed rest with head elevation no greater than 30 187 (227)

The nurse is preparing to care for client who has returned to the nursing unit following cardiac catheterization performed through the femoral artery. The nurse checks the HCP prescription and plans to allow which client position or activity following procedure? 1. Best rest in high Fowler-s position 2. Best rest with bathroom privileges only 3. Best rest with head elevation at 60 4. Bed rest with head elevation no greater than 30

4. A multiparous client who delivered a large baby after oxytocin induction 336 (380)

The nurse is preparing to care for four assigned clients. Which client is at highest risk for hemorrhage? 1. A primiparous client who delivered 4 hours ago 2. A multiparous client who delivered 6 hours ago 3. A primiparous client who delivered 6 hours ago and had epidural anesthesia 4. A multiparous client who delivered a large baby after oxytocin induction

4. Take a deep breath, hold it, and bear down 88 (144)

The nurse is preparing to change the PN solution bag and tubing. The client's central venous like is located in the right subclavian vein. The nurse asks the client to take which essential action during the tubing change? 1. Breathe normally 2. Turn head to the right 3. Exhale slowly and evenly 4. Take a deep breath, hold it, and bear down

2. Obtains a different bottle of solution 93 (145)

The nurse is preparing to hang fat emulsion and notes that fat globules are visible at the top of the solution. The nurse should take which action? 1. Rolls the bottle of solution gently 2. Obtains a different bottle of solution 3. Shakes the bottle of solution vigorously 4. Runs the bottle of solution under warm water

3. Electronic infusion pump 97 (145)

The nurse is preparing to hang the first bag of PN solution via the central line of an assigned client. The nurse should obtain which most essential piece of equipment before hanging the solution? 1. Urine test strips 2. Blood glucose meter 3. Electronic infusion pump 4. Noninvasive blood pressure monitor

2. Contact the electrical maintenance department for assistance 131 (183)

The nurse is preparing to initiate an IV line containing a high dose of potassium chloride and plans to use an IV infusion pump. The nurse brings the pump to the bedside, prepares to plug the pump cord into the wall, and notes that no receptacle is available in the wall socket. The nurse should take which action? 1. Initiate the IV line without the use of a pump 2. Contact the electrical maintenance department for assistance 3. Plug in the pump cord in the available plug above the room sink 4. Use an extension cord from the nurse's lounge for the pump plug

3. High-Fowler's 188 (227)

The nurse is preparing to insert an NG tube into a client. The nurse should place the client in which position for insertion? 1. Right side 2. Low-Fowler's 3. High-Fowler's 4. Supine with the HOB flat

1,2,3,5 189 (242)

The nurse is preparing to instill meds into a client's NG tube. Which actions should the nurse take before instilling the medication? 1. Check the residual volume 2. Aspirate the stomach contents 3. Turn off the suction to the NG tube 4. Remove the tube and place it in the other nostril 5. Test the stomach contents for a pH of less than 3.5

3. A 90 woman who has advanced Parkinson's disease 226 (274)

The nurse is providing an educational session to new employees, and the topic is abuse of the older client. The nurse helps the employees identify which client as most typically a victim of abuse? 1. A 75 man who has moderate HTN 2. a 68 man who has newly diagnosed cataracts 3. A 90 woman who has advanced Parkinson's disease 4. a 70 woman who has early diagnosed Lyme disease

1. Continue with the instruction, verifying client understanding.

The nurse is providing discharge instruction to a Chinese American client regarding prescribed dietary modifications. During the teaching session, the client continuously turns away from the nurse. The nurse should implement which best action? 1. Continue with the instruction, verifying client understanding. 2. Walk around the client so that the nurse constantly faces the client. 3. Give the client a dietary booklet and return later to continue with the instruction. 4. Tell the client about the importance of the instructions for the maintenance of health care.

3. Swimming 375 (426)

The nurse is providing home care instruction to the parents of a 10 year old child with hemophilia. Which sport activity should the nurse suggest for this child? 1. Soccer 2. Basketball 3. Swimming 4. Field hockey

3. They respond to low-pitch tones 225 (274)

The nurse is providing instruction to the UAP regarding care of an older client with hearing loss. Which should the nurse tell the UAP about older clients with hearing loss? 1. They are often distracted 2. They have middle ear changes 3. They respond to low-pitch tones 4. They develop moist cerumen production

4. I should wash my nipples daily with soap and water 332 (380)

The nurse is providing instructions about measures to prevent postpartum mastitis to a client who is breast-feeding her newborn. Which client statement would indicate a need for further instruction? 1. I should breast-feed every 2-3 hours 2. I should change the breast pads frequently 3. I should wash my hands well before breast-feeding 4. I should wash my nipples daily with soap and water

4. I should apply heat packs to the hemorrhoids to help them shrink 273 (323)

The nurse is providing instructions regarding treatment of hemorrhoids to a client who is in the second trimester of pregnancy. Which statement by the client indicates a need for further instruction? 1. I should avoid straining during BM 2. I can gently replace the hemorrhoids into the rectum 3. I can apply ice packs to the hemorrhoids to reduce the swelling 4. I should apply heat packs to the hemorrhoids to help them shrink

2. I should not sleep on my right side 183 (227)

The nurse is providing instructions to a client and the family regarding home care after right eye cataract removal. Which statement by the client would indicate and understanding of the instructions? 1. I should not sleep on my left side 2. I should not sleep on my right side 3. I should not sleep with my head elevated 4. I should not wear my glasses at any time

2. Wash the breast with warm water and keep them dry 274 (324)

The nurse is providing instructions to a client in the first trimester of pregnancy regarding measures to assist in reducing breast tenderness. Which instruction should the nurse provide? 1. Avoid wearing a bra 2. Wash the breast with warm water and keep them dry 3. Wear tight-fitting blouses or dresses to provide support 4. Wash the nipples and areolar area daily with soap, and massage the breasts with lotion

4. I should drink adequate fluids and increase my intake of high-fiber foods 286 (341)

The nurse is providing instructions to a maternity client with a history o cardiac disease regarding appropriate dietary measures. Which statement, if made by the client, indicates an understanding of the information provided by the nurse? 1. I should increase my sodium intake during pregnancy 2. I should lower my blood volume by limiting my fluids 3. I should maintain a low-calorie diet to prevent any weight gain 4. I should drink adequate fluids and increase my intake of high-fiber foods

3. An informed consent needs to be signed before the procedure 262 (322)

The nurse is providing instructions to a pregnant client who is scheduled for an amniocentesis. What instruction should the nurse provide? 1. Strict bed rest is required after the procedure 2. Hospitalization is necessary for 24 hours after the procedure 3. An informed consent needs to be signed before the procedure 4. A fever is expected after the procedure because of the trauma to the abdomen

1. You need to bottle-feed your newborn 277 (340)

The nurse is providing instructions to a pregnant client with HIV infection regarding care to the newborn after delivery. The client asks the nurse about the feeding options that are available. Which response should the nurse make to the client? 1. You need to bottle-feed your newborn 2. You will need to feed your newborn by NG tube 3. You will be able to breast-feed for 6 months and then you will need to switch to bottle-feeding 4. You will be able to breast-feed for 9 months and then you will need to switch to bottle-feeding

4. A cesarean section will be necessary if vaginal lesions are present at the time of labor 255 (310)

The nurse is providing instructions to a pregnant client with genital herpes about the measures that are needed to protect the fetus. Which instruction should the nurse provide to the client? 1. Total abstinence from sexual intercourse is necessary during the entire pregnancy 2. Stiz baths need to be taken every 4 hours while awake if vaginal lesions are present 3. Daily administration of acyclovir is necessary during the entire pregnancy 4. A cesarean section will be necessary if vaginal lesions are present at the time of labor

4. Decreased lean body mass and decreased glomerular filtration rate 222 (274)

The nurse is providing med instructions to an older client who is taking digoxin daily. The nurse notes that which age-related body change could place the client at risk for digoxin toxicity? 1. Decreased muscle strength and loss of bone density 2. Decreased cough efficiency and decreased vital capacity 3. Decreased salivation and decreased gi motility 4. Decreased lean body mass and decreased glomerular filtration rate

1,2,3,6 324 (373)

The nurse is providing postpartum instructions to a client who will be breast-feeding her newborn. The nurse determines that the client has understood the instructions if she makes which statements? Select all that apply 1. I should wear a bra that provides support 2. Drinking alcohol can affect my milk supply 3. The use of caffeine can decrease my milk supply 4. I will start my estrogen birth control pills again as soon as i get home 5. I know if my breasts get engorged i will limit my breast-feeding and supplement the baby 6. I plan on having bottled water available in the refrigerator so i can get additional fluids easily

1. The skin Insensible losses occur through the skin or lungs. Sensible losses are from urination, wound drainage, and GI losses 48 (99)

The nurse is reading a HCP progress note in the client's record and reads that the HCP has documented "insensible fluid loss of approximately 800mL daily." The nurse interprets that this type of fluid loss can occur through which outcome? 1. The skin 2. UO 3. Wound drainage 4. The GI tract

1. Predinisone Predinisone is a corticosteroid used to help decrease stress on the body 178 (218)

The nurse is reviewing a HCP prescription sheet for a pre-op client that states that the client must be NPO after midnight. The nurse would telephone the HCP to clarify that which medication should be given to the client and not withheld? 1. Predinisone 2. Ferrous sulfate 3. Cyclobenzaprine 4. Conjugated estrogen

4. Placing the client in a semiprivate room at the end of the hallway 135 (184)

The nurse is reviewing a plan of care for a client with an internal radiation implant. Which intervention if noted in the plan indicates the need for revision of the plan? 1. Wearing gloves when emptying the client's bedpan 2. Keeping all linens in the room until the implant is removed 3. Wearing a lead apron when providing direct care to the client 4. Placing the client in a semiprivate room at the end of the hallway

1,6 382 (427)

The nurse is reviewing the HCP prescriptions for a child with sickle cell anemia who was admitted to the hosppital for the treatment of vaso-occlusive crisis. Which prescriptions documented in the child's record should the nurse question? Select all that apply 1. Restrict fluid intake 2. Position for comfort 3. Avoid strain on painful joints 4. Apply nasal oxygen at 2L 5. Provide a high-calorie, high-protein diet 6. Give meperidine 25mg IV q4hrs

3. Perform a vaginal exam every shift 314 (365)

The nurse is reviewing the HCP prescriptions for a client admitted for premature rupture of the membranes. Gestational age of the fetus is determined to be 37 weeks. Which prescription should the nurse question? 1. Monitor FHR continuously 2. Monitor maternal VS frequently 3. Perform a vaginal exam every shift 4. Administer ampicillin 1g as an iv piggyback every 6 hours

3. 1 cm above the ischial spine Measured in cm 294 (355)

The nurse is reviewing the record of a client in the labor room and notes that the HCP has documented that the fetal presenting part is at the -1 station. This documented finding indicates that the fetal presenting part is located at which area? 1. 1 inch below the coccyx 2. 1 inch below the iliac crest 3. 1 cm above the ischial spine 4. 1 fingerbreadth below the symphysis pubic

1. Softening of the cervix 256 (310)

The nurse is reviewing the record of a client who has just been told that a pregnancy test is positive. The HCP has documented the presence of Goodell's sign. This finding is most closely associated with which characteristic? 1. Softening of the cervix 2. The presence of fetal movement 3. The presence of human chorionic gonadotropin in the urine 4. A soft blowing sound that corresponds to the maternal pulse during auscultation of the fetus

4. My contractions will increase in duration and intensity 302 (355)

The nurse is reviewing true and false labor signs with a multiparous client. The nurse determines that the client understands the signs of true labor if she makes which statement? 1. I wont be in labor until my baby drops 2. My contractions will be felt in my abdominal area 3. My contractions will not be painful if i walk around 4. My contractions will increase in duration and intensity

4. The object may be forced back father into the throat 162 (204)

The nurse is teaching CPR to a group of nursing students. The nurse asks the student to describe the reason why blind finger sweps are avoided in infants. The nurse determines that the student understands this reason if the student makes which statement? 1. The object may have been swallowed 2. The infant may bite down on the finger 3. The mouth is too small to see the object 4. The object may be forced back father into the throat

3. Overhead trapeze

The nurse is teaching a client in skeletal leg traction about measures to increase bed motility. Which item would be most helpful to the client. 1. Television 2. Fracture bedpan 3. Overhead trapeze 4. Reading materials

4. Oranges and dark green leafy vegetables 77 (135)

The nurse is teaching a client who has iron deficiency anemia about foods she should include in her diet. The nurse determines that the client understands the dietary modifications if she selects which items from her menu? 1. Nuts and milk 2. Coffee and tea 3. Cooked rolled oats and fish 4. Oranges and dark green leafy vegetables

1. The diet should include additional fluids 325 (373)

The nurse is teaching a postpartum client about breast-feeding. Which instruction should the nurse include? 1. The diet should include additional fluids 2. Prenatal vitamins should be discontinued 3. Soap should be used to cleanse the breasts 4. Birth control measures are unnecessary while breast-feeding

2. Tests the 6 cardinal positions of gaze 237 (294)

The nurse is testing the extraocular movements in a client to assess for muscle weakness in the eyes. The nurse should implement which assessment technique to assess for muscle weakness in the eye? 1. Tests the corneal reflexes 2. Tests the 6 cardinal positions of gaze 3. Tests visual acuity, using a Snellen eye chart 4. Tests sensory function by asking the client to close eyes and then lightly toughing the forehead, cheeks, and chin

4. Confront the UAP to encourage verbalization of feelings regarding the change

The nurse manager has implemented a change in the method of the nursing delivery system from functional to team nursing. An UAP is resistant to the change and is not taking an active part in facilitating the process of change. Which is the best approach in dealing with the UAP? 1. Ignore the resistance 2. Exert coercion on the UAP 3. Provide a positive reward system for the UAP 4. Confront the UAP to encourage verbalization of feelings regarding the change

1. Anorexia

The nurse monitors a client receiving digoxin (Lanoxin) for which early manifestation of digoxin toxicity. 1. Anorexia 2. Facial pain 3. Photophobia 4. Yellow color perception

1. Discontinue the IV

The nurse notes blanching, coolness, and edema at the peripheral intravenous site. On the basis of these findings, the nurse should implement which action? 1. Discontinue the IV 2. Apply a warm compress 3. Check for a blood return 4. Measure the area of infiltration

3. A physical obstruction to the transmission of sound waves 235 (294)

The nurse notes documentation that a client has conductive hearing loss. The nurse understand that this type of hearing loss is caused by which problem? 1. A defect in the cochlea 2. A defect in the 8th cranial nerve 3. A physical obstruction to the transmission of sound waves 4. A defect in the sensory fibers that lead to the cerebral cortex

1. Rhythmic respiration with periods of apnea 234 (294)

The nurse notes documentation that a client is exhibiting Cheyne-Stokes respiration. On assessment of the client, the nurse should expect to note which finding? 1. Rhythmic respiration with periods of apnea 2. Regular rapid and deep, sustained respirations 3. Totally irregular respirations in rhythm and depth 4. Irregular respirations with pauses at the end of inspiration and expiration

2. Abnormal palmar creases 348 (399)

The nurse notes hypotonia, irritability, and a poor sucking reflex in a full-term newborn on admission to the nursery. The nurse suspects fetal alcohol syndrome and is aware that which additional sign would be consistent with this syndrome? 1. Length of 19 inches 2. Abnormal palmar creases 3. Birth weight of 6lb 14ox 4. Head circumference appropriate for gestational age

1,2,4,6 59 (112)

The nurse notes that a client's ABG results reveal a pH of 7.50 and a PCO2 of 30. The nurse monitors the client for which clinical manifestations associated with these ABG results? Select all that apply 1. Nausea 2. Confusion 3. Bradypnea 4. Tachycardia 5. Hyperkalemia 6. Lightheadedness

1. Phlebitis of the vein 108 (160)

The nurse notes that the site of a client' IV catheter is reddened, warm, painful, and slightly edematous proximal to the insertion point of the IV catheter. After taking appropriate steps to care for the client, the nurse should document in the medical record that the client experienced which condition? 1. Phlebitis of the vein 2. Infiltration of the IV site 3. Hypersensitivity to the IV solution 4. Allergic reaction to the IV catheter material

2. Safely securing the safety device straps to the side rails 132 (183)

The nurse obtains a prescription from a HCP to restrain a client and instructs an UAP to apply the safety device to the client. Which observation by the nurse indicates unsafe application of the safety device by the UAP? 1. Placing a safety know in the safety device straps 2. Safely securing the safety device straps to the side rails 3. Applying safety device straps that to not tighten when force is applied against them 4. Securing so that two fingers can slide easily between the safety device and the client's skin

3. Start chest compressions 156 (203)

The nurse on the day shift walks into a client's room and finds the client unresponsive. The client is not breathing and does not have a pulse and the nurse immediately calls out for help. Which is the next nursing action? 1. Open the airway 2. Give the client oxygen 3. Start chest compressions 4. Ventilate with a mouth-to-mask device

3. Respiratory acidosis 62 (112)

The nurse plans care for a client with COPD, understanding that the client is most likely to experience what type of acid-base imbalance? 1. Metabolic acidosis 2. Metabolic alkalosis 3. Respiratory acidosis 4. Respiratory alkalosis

4. Newborns are deficient in vitamin K, and this injection prevents your newborn from bleeding 355 (399)

The nurse prepares to administer the Vitamin K injection to the newborn, and the mother asks the nurse why her infant needs the injection. What best response should the nurse provide? 1. Your newborn needs Vitamin K to develop immunity 2. The vitamin K will protect your newborn from being jaundiced 3. Newborns have sterile bowels, and vitamin K promotes the growth of bacteria in the bowel 4. Newborns are deficient in vitamin K, and this injection prevents your newborn from bleeding

3. Gloves, mask, gown, and goggles

The nurse prepares to care for a client on contact precautions who has a hospital-acquired infection caused by MRSA. The client has an abdominal wound that requires irrigation and has a trach attached to a ventilator, which requires frequent suctioning. The nurse should assemble which necessary protective items before entering the client's room? 1. Gloves and a gown 2. Gloves, mask, and goggles 3. Gloves, mask, gown, and goggles 4. Gloves, gown, and show protectors

2. I need to restrict my activity while this catheter is in place 113 (160)

The nurse provides a list of instructions to a client being discharged to home with a PICC. The nurse determines that the client needs further instructions if the client made which statement? 1. I need to wear a Medic-Alert tag or bracelet 2. I need to restrict my activity while this catheter is in place 3. I need to have a repair kit available in the home for use if needed 4. I need to keep the insertion site protected when in the shower or bath

2,3,4,6 Peas and Cauliflower are high in magnesium 40 (99)

The nurse provides instruction to a client with a low potassium level about the foods that are high in potassium and tells the client to consume which foods? Select all that apply 1. Peas 2. Raisins 3. Potatoes 4. Cantaloupe 5. Cauliflower 6. Strawberries

4. "If my pulse rate drops below 60 bpm, I should let my HCP know."

The nurse provides medication instructions to a client about digoxin (Lanoxin). Which statement by the client indicates an understanding of its adverse effects? 1. "Blurred vision is expected." 2. "If I am nauseated or vomiting, I should stay on liquids and take some liquid antacids." 3. "This medication may cause headache and weakness but that is nothing to worry about." 4. "If my pulse rate drops below 60 bpm, I should let my HCP know."

1. Assess the patency of the airway ABC's 177 (218)

The nurse receives a telephone call from the post anesthesia care unit stating that a client is being transferred to the surgical unit. The nurse plans to take which action first on arrival of the client? 1. Assess the patency of the airway 2. Check tubes or drains for patency 3. Check the dressing to assess for bleeding 4. Assess the vital signs to compare with pre-op measurements

1. U waves 38 (99)

The nurse reviews a client's electrolyte lab report and notes that the potassium level is 2.5. Which pattern would the nurse note on the EKG as a result of the lab value? 1. U waves 2. Absent P waves 3. Elevated T waves 4. Elevated ST segment

1. Alcoholism The other options are consistent with increased phosphate levels. 47 (99)

The nurse reviews a client's lab report and notes that the client's serum phosphorus level is 2. Which condition most likely caused this phosphorus level? 1. Alcoholism 2. Renal insufficiency 3. Hypoparathyroidism 4. Tumor lysis syndrome

2. Respiratory alkalosis, compensated 53 (111)

The nurse reviews the ABG results of a client and notes the following: pH 7.45, PCO2 of 30, HCO3 of 20. The nurse analyzes these results as indicating which condition? 1. Metabolic acidosis, compensated 2. Respiratory alkalosis, compensated 3. Metabolic alkalosis, uncompensated 4. Respiratory acidosis, uncompensated

1. pH 7.25, PCO2 50 60 (112)

The nurse reviews the blood gas results of a client with atelectasis. The nurse analyzes the results and determines that the client is experiencing respiratory acidosis. Which result validates the nurse's findings? 1. pH 7.25, PCO2 50 2. pH 7.35, PCO2 40 3. pH 7.50, PCO2 52 4. pH 7.52, PCO2 28

4. Tall peaked T waves 44 (99)

The nurse reviews the electrolyte results of an assigned client and notes that the potassium level is 5.7. Which finding would the nurse expect to note on the EKG as a result of the lab value? 1. ST depression 2. Inverted T waves 3. Prominent U wave 4. Tall peaked T waves

4. Around-the-clock dosing gives better pain relief than as-needed dosing.

The nurse should consider which factor when planning opioid pain relief? 1. Not all pain is real pain. 2. Opioid analgesics are highly addictive. 3. Opioid analgesics can cause tachycardia. 4. Around-the-clock dosing gives better pain relief than as-needed dosing.

2. Your type of pelvis is the most favorable for labor and birth 249 (304)

The nurse should include which statement to a pregnant client found to have a gynecoid pelvis? 1. You type of pelvis has a narrow pubic arch 2. Your type of pelvis is the most favorable for labor and birth 3. Your type of pelvis is a wide pelvis, but has a short diameter 4. You will need a cesarean section because this type of pelvis is not favorable for a vaginal delivery

1,5

The nurse should use which guidelines to plan delegation and assignment-making activities? Select all that apply 1. Ensuring client safety 2. Requests from the staff 3. The clustering of the rooms on the unit 4. The number of anticipated client discharges 5. Client needs and workers' needs and abilities

4. Each rescue breath should be given over 1 second and should produce a visible chest rise 158 (204)

The nurse understand that which is a correct guideline for adult CPR for a HCP? 1. One breath should be given for every 5 compressions 2. Two breaths should be given for every 15 compressions 3. Initially, two quick breaths should be given as rapidly as possible 4. Each rescue breath should be given over 1 second and should produce a visible chest rise

1. Expiration date 127 (170)

The nurse who is about to begin a blood transfusion knows that blood cells start to deteriorate after a certain period of time. Which item is important to check regarding the age of blood cells before the transfusion is begun? 1. Expiration date 2. Presence of clots 3. Blood group and type 4. Blood identification number

3. Call the nursing supervisor

The nurse who works on the night shift enters the medication room and finds a co-worker with a tourniquet wrapped around the upper arm. The co-worker is about to insert a needle, attached to a syringe containing a clear liquid, into the antecubital area. Which is the most appropriate action by the nurse. 1. Call security 2. Call the police 3. Call the nursing supervisor 4. Lock the co-worker in the medication room until help is obtained

3. Jaw thrust maneuver

The nurse witnesses a neighbor's husband sustain a fall from the roof of his house. The nurse rushes to the victim and determines the need to open the airway. The nurse opens the airway in this victim by using which method? 1. Flexed position 2. Head tilt-chin lift 3. Jaw thrust maneuver 4. Modified head tilt-chin lift

3. Activate the emergency response system 163 (204)

The nurse witnesses the collapse of a victim in her neighborhood and suspects cardiac arrest. Which action should the nurse take first? 1. Initiate rescue breathing 2. Being giving chest compression 3. Activate the emergency response system 4. Obtain an automated external defibrillator

4. WBC count 125 (170)

The nurse, listening to the morning report, learns that an assigned client received a unit of granulocytes the previous evening. The nurse makes a note to assess the results of which daily serum lab study to assess the effectiveness of the transfusion? 1. HCT level 2. Erythrocyte count 3. HGB level 4. WBC count

3. 2 inches 165 (204)

The nurseis performing CPR on an adult client. When performing chest compressions, the nurse should depress the sternum by how many inches? 1. 3/4 inch 2. 1 inch 3. 2 inches 4. 3 inches

4. Processed oat cereals Processed foods are high in sodium 41 (99)

The nurseis reviewing lab results and notes that a client's sodium level is 150. The nurse reports the sodium level to the HCP and the HCP prescribes dietary instructions based on the sodium level. Which food item does the nurse instruct the client to avoid? 1. Peas 2. Nuts 3. Cauliflower 4. Processed oat cereals

1. This stage is associated with toilet training 205 (253)

The nursing instructor ask a nursing student to present a clinical conference to peers regarding Freud's psychosexual stages of development, specifically the anal stage. The student plans the conference, knowing that which characteristic relates to this stage of development? 1. This stage is associated with toilet training 2. This stage is characterized by the gratification of self 3. This stage is characterized by tapering off conscious biological and sexual urges 4. This stage is associated with pleasurable and conflicting feeling about the genital organs

1. The child has the ability to think abstractly 205 (253)

The nursing instructor asks a nursing student to describe the formal operations stage of Piaget's cognitive developmental theory. The correct response by the nursing student is which statement? 1. The child has the ability to think abstractly 2. The child begins to understand the environment 3. The child is able to classify, order, and sort facts 4. The child learns to think in terms of past, present, and future

1. I will flush the eyes after instilling the ointment 359 (410)

The nursing instructor asks a nursing student to describe the procedure for administering erythromycin ointment to the eyes of a newborn. Which student statement indicates that further teaching is needed? 1. I will flush the eyes after instilling the ointment 2. I will clean the newborn's eyes before instilling ointment 3. I need to administer the eye ointment within 1 hour after delivery 4. I will instill the eye ointment into each of the newborn's conjunctival sacs

2. Arrange for an interpreter to translate

When communicating with a client who speaks a different language, which best practice should the nurse implement? 1. Speak loudly and slowly 2. Arrange for an interpreter to translate 3. Speak to the client and family together 4. Sand close to the client and speak loudly

3. Notify the HCP Clots larger than 1 cm are abnormal and need to be reported 328 (373)

When performing a postpartum assessment on a client, a nurse notes the presence of clots in the lochia. The nurse examines the clots and notes that they are larger than 1 cm. Which nursing action is most appropriate? 1. Document these findings 2. Reassess the client in 2 hours 3. Notify the HCP 4. Encourage increased intake of fluids

2. Booster seat 211 (266)

Which care safety device should be used for a child who is 8 years old and 4 feet tall? 1. Seat belt 2. Booster seat 3. Rear-facing convertible seat 4. Front-facing convertible seat

1,2,4,5

Which client(s) has a high risk of obesity and diabetes mellitus? Select all that apply 1. a 40-yr-old Latino American man 2. A 45-yr-old Native American woman 3. A23-yr-old Asian American woman 4. A 35-yr-old Hispanic American man 5. A 40-yr-old African American woman

3. It is the way the baby gets food and oxygen 250 (304)

Which explanation should the nurse provide to the prenatal client about eh purpose of the placenta? 1. It cushions and protects the baby 2. It maintains the temperature of the baby 3. It is the way the baby gets food and oxygen 4. It prevents all antibodies and viruses from passing to the baby

1,2,5

Which identifies accurate nursing documentation notations? Select all that apply 1. The client slept through the night. 2. Abdominal wound dressing is dry and intact without drainage. 3. The client seemed angry when awakened for vital sign measurement. 4. The client appears to become anxious when it is time for respiratory treatments. 5. The client's left lower medial leg wound is 3 cm in length without redness, drainage, or edema.

1,4,5 220 (267)

Which interventions are appropriate for the care of an infant? Select all that apply 1. Provide swaddling 2. Talk in a loud voice 3. Provide the infant with a bottle of juice at nap time 4. Hang mobiles with black and white contrast designs 5. Caress the infant while bathing or during diaper changes 6. Allow the infant to cry for at least 10 minutes before responding

3. Sweet and sour chicken with rice and vegetables, mixed fruit, juice

Which meal tray should the nurse deliver to a client of Orthodox Judaism faith who follows a kosher diet? 1. Pork roast, rice, vegetables, mixed fruit, milk 2. Crab salad on a croissant, vegetables with dip, potato salad, milk 3. Sweet and sour chicken with rice and vegetables, mixed fruit, juice 4. Noodles and cream sauce with shrimp and vegetables, salad, mixed fruit, iced tea

2. Determine whether the client has an allergy to eggs 90 (144)

Which nursing action is essential prior to initiating a new prescription for 500mL of fat emulsion to infuse at 50mL/hr? 1. Ensure the client does not have diabetes 2. Determine whether the client has an allergy to eggs 3. Add regular insulin to the fat emulsion, using aseptic technique 4. Contact the HCP to have a central line inserted for fat emulsion infusion

2,3,4

Which specific nursing interventions are implemented in the care of a child with leukemia who is at risk for infection? Select all that apply 1. Maintain the child in a semi-private room 2. Reduce exposure to environmental organisms 3. Use strict aseptic technique for all procedures 4. Ensure that anyone entering the child's room wears a mask 5. Apply firm pressure to a needle stick area for at least 10 minutes

4. I will ask the nurse to attend to my infant if i am napping and my husband is not here 354 (399)

Which statement reflets a new mother's understand of the teaching about prevention of newborn abduction? 1. I will place my baby's crib close to the door 2. Some health care personnel wont have name badges 3. It's OK to allow the UAP to carry my newborn to the nursery 4. I will ask the nurse to attend to my infant if i am napping and my husband is not here

2. Grasp the retention sutures to spread the opening 196(243)

While changing the tapes on a trach tube, the client coughs and the tube is dislodged. Which is the initial nursing action? 1. Call the HCP to reinsert the tube 2. Grasp the retention sutures to spread the opening 3. Call the respiratory therapy department to reinsert the trach 4. Cover the trach site with a sterile dressing to prevent infection

3. Gentle blowing or swooshing noise 236 (294)

While performing a cardiac assessment on a client with an incompetent heart valve, the nurse auscultates a murmur. Which describes the sound of a heart murmur? 1. Lub-dub 2. Scratchy, leathery heart noise 3. Gentle blowing or swooshing noise 4. Abrupt, high pitched snapping noise

4. Suggest appropriate resources to the client and daughter,such as respite care and a senior citizens' center 228 (274)

The visiting nurse observes that the older male client is confined by his daughter-in-law to his room. When the nurse suggests that he walk to the den and join the family, he says, "I'm in everyone's way; my daughter-in-law needs me to stay here." Which is the most important action for the nurse to take? 1. Say to the daughter-in-law, "Confining your father to his room in inhumane." 2. Suggest to the client and daughter that they consider a nursing home for the client 3. Say nothing, because it is best for the nurse to remain neutral and wait to be asked for help 4. Suggest appropriate resources to the client and daughter,such as respite care and a senior citizens' center

3. IV infusion of factor VIII 379 (426)

A 10 year old child with hemophilia A has slipped on the ice and bumped his knee. The nurse should prepare to administer which prescription? 1. Injection of factor X 2. IV infusion of iron 3. IV infusion of factor VIII 4. IM injection of iron using the Z track method

4. Allow the child to interact with others in his or her same age group 210 (266)

A 16-year old is admitted to the hospital for acute appendicitis and an appendectomy is performed. Which nursing intervention is most appropriate to facilitate normal growth and development postop? 1. Encourage the child to rest and read 2. Encourage the parents to room in with the child 3. Allow the family to bring in the child's favorite computer game 4. Allow the child to interact with others in his or her same age group

3. Bone marrow biopsy

A 4 year old is admitted to the hospital for abdominal pain. The mother reports that the child has been pale and excessively tired and is bruising easily. On physical exam, lymphadenopathy and hepatosplenomegaly are noted. Diagnostic studies are being performed on the child because acute lymphocytic leukemia is suspected. The nurse understands that which diagnostic study should confirm this diagnosis? 1. Platelet count 2. Lumbar puncture 3. Bone marrow biopsy 4. WBC count

1. Encourage the child's parents to stay with the child 209 (266)

A 4-year child diagnosed with leukemia is hospitalized for chemotherapy. The child is fearful of the hospitalization. Which nursing intervention should be implemented to alleviate the child's fears? 1. Encourage the child's parents to stay with the child 2. Encourage play with other children of the same age 3. Advise the family to visit only during the scheduled visiting hours 4. Provide a private room, allowing the child to bring favorite toys from home

3. The probe that will be inserted into the vagina will be covered with a disposable cover and coated with a gel 268 (323)

A HCP has prescribed transvaginal ultrasonography for a client in the first trimester of pregnancy and the client asks the nurse about the procedure. How should the nurse respond to the client? 1. The procedure takes about 2 hours 2. It will be necessary to drink 1 to 2 quarts of water before the procedure 3. The probe that will be inserted into the vagina will be covered with a disposable cover and coated with a gel 4. Gel is spread over the abdomen, and a round disk transducer will be moved over the abdomen to obtain a picture

4. Sterile 2x2 gauze 110 (160)

A HCP has written a prescription to discontinue an IV line. The nurse should obtain which item from the unit supply area for applying pressure to the site after removing the IV catheter? 1. Elastic wrap 2. Betadine swab 3. Adhesive bandage 4. Sterile 2x2 gauze

8 hours 152 (194)

A HCP prescribes 1000mL D5W to infuse at a rate of 125mL/hr. The nurse determines that it will take how many hours ofr 1 L to infuse

1. Supine position with a wedge under the right hip 297 (355)

A client in labor is transported to the delivery room and prepared for a cesarean delivery. After the client is transferred to the delivery room table,the nurse should place the client in which position? 1. Supine position with a wedge under the right hip 2. Trendelenburg's position with the legs in stirrups 3. Prone position with the legs separated and elevated 4. Semi-Fowler's position with a pillow under the knees

2. Betamethasone 360 (410)

A client in preterm labor (31 weeks) who is dilated to 4cm has been started on mag sulfate and contractions have stopped. If the client's labor can be inhibited for the next 48 hours, the nurse anticipates a prescription for which med? 1. Nalbuphine 2. Betamethasone 3. Rh immune globulin 4. Dinoprostone

1. What can i do for you 279 (340)

A stillborn baby was delivered in the birthing suite a few hours ago. After the delivery, the family remained together, holding and touching the baby. Which statement by the nurse would further assist the family in their initial period of grief? 1. What can i do for you 2. Now you have an angel in heaven 3. Don't worry, there is nothing you could have done to prevent this from happening 4. We will see to it that you have an early discharge so that you don't have to be reminded of this experience

2. Support the mother in her reaction to the newborn infant 311 (364)

After a precipitous delivery, the nurse notes that the new mother is passive and only touches her newborn infant briefly with her fingertips. What should the nurse do to help the woman process the delivery? 1. Encourage the mother to breast-feed soon after birth 2. Support the mother in her reaction to the newborn infant 3. Tell the mother that it is important to hold the newborn infant 4. Document a complete account of the mother's reaction on the birth record

3. Foods considered to be yin

An Asian American client is experiencing a fever. The nurse recognizes that the client is likely to self-treat the disorder, using which method? 1. Prayer 2. Magnetic therapy 3. Foods considered to be yin 4. Foods considered to be yang

3. Iron deficiency anemia HGB 12-15 75 (124)

An adult female client has a HGB level of 10.8. The nurse interprets that this result is most likely caused by which condition notes in the client's history? 1. Dehydration 2. HF 3. Iron deficiency anemia 4. COPD

25 drops per minute 151 (193)

Cefuroxime sodium, 1g in 50mL NS, is to be administered over 30 minutes. The drop factor is 15 drops /mL. The nurse sets the flow rate at how many drops per minute?

4. Gloves, gown, goggles, and face shield 136 (184)

Contact precautions are initiated for a client with a health care-associated infection caused by methicillin-resistant Staphylococcus anureus. The nurse prepares to provide colostomy care and should obtain which protective items to preform this procedure? 1. Gloves and gown 2. Gloves and goggles 3. Gloves, gown and shoe protectors 4. Gloves, gown, goggles, and face shield

4. Administer oxygen 8-10L via face mask 316 (365)

Fetal distress is occurring with a laboring client. As the nurse prepares the client for a cesarean birth, what is the most important nursing action? 1. Slow the IV flow rate 2. Place the client in a high Fowler's position 3. Continue the oxytocin drip if infusing 4. Administer oxygen 8-10L via face mask

2. Place the client in the high-Fowler's position 124 (170)

Following infusion of a unit of PRBC, the client has developed new onset of tachycardia, bounding pulses, crackles, and wheezes. Which action should the nurse implement first? 1. Maintain bed rest with legs elevated 2. Place the client in the high-Fowler's position 3. Increase the rate of infusion of IV fluids 4. Consult with the HCP regarding initiation of oxygen therapy

2. Slander

Nursing staff members are sitting in the lounge taking their morning break. An UAP tells the group that she thinks that the unit secretary has AIDS and proceeds to tell the nursing staff that the secretary probably contracted the disease from her husband, who is supposedly a drug addict. Which legal tort has the UAP violated? 1. Libel 2. Slander 3. Assault 4. Negligence

4. Apply the lotion to cool, dry skin at least 30 minutes after bathing 368 (419)

Permethrin is prescribed for a child with a diagnosis of scabies. The nurse should give which instruction to the parents regarding the use of this treatment? 1. Apply the lotion to areas of the rash only 2. Apply the lotion and leave it on for 6 hours 3. Avoid putting clothes on the child over the lotion 4. Apply the lotion to cool, dry skin at least 30 minutes after bathing

1,2,4,6 Calcium 8.6-10 Magnesium 1.6-2.6 Phosphorus 2.5-4.5 Neutrophils 1800-7800 Creatinine 0.6-1.3 WBC 4500-11000 69 (124)

Several lab tests are prescribed for a client, and the nurse reviews the results of the tests. Which abnormal lab test results should the nurse report? Select all that apply 1. Calcium 7 2. Magnesium 1 3. Phosphorus 3.6 4. Neutrophils 1000 5. Creatinine 1 6. WBC 3000

3. A client who requires urine specimen collections

The RN is planning the client assignment for the day. Which is the most appropriate assignment for a UAP? 1. A client requiring a colostomy irrigation 2. A client receiving continuous tube feedings 3. A client who requires urine specimen collections 4. A client with difficulty swallowing food and fluids

3. Place the tube at the tip of the nose and measure by extending the tube to the earlobe and then down to the xiphoid process 193 (242)

The RN is preparing to insert a NG tube in an adult client. To determine the accurate measurement of the length of the tube to be inserted, the nurse should take which action? 1. Mark the tube at 10 inches 2. Mark the tube at 32 inches 3. Place the tube at the tip of the nose and measure by extending the tube to the earlobe and then down to the xiphoid process 4. Place the tube at the tip of the nose and measure by extending the tube to the earlobe and then down to the top of the sternum

4. The client complains of headache and blurred vision 278 (340)

The home care nurse visits a pregnant client who has a diagnosis of mild preeclampsia. Which assessment finding indicates a worsening of the preeclampsia and the need to notify the HCP? 1. Urinary output has increased 2. Dependent edema has resolved 3. BP reading is at the prenatal baseline 4. The client complains of headache and blurred vision

2. Allow the bottle if it contains water 217 (266)

The mother of a 3 year old is concerned because her child is still insisting on a bottle at nap time and at bedtime. Which is the most appropriate suggestions to the mother? 1. Allow the bottle if it contains juice 2. Allow the bottle if it contains water 3. Do not allow the child to have the bottle 4. Allow the bottle during naps but not at bedtime

3. FHR pattern 303 (356)

Which assessment finding following an amniotomy should be conducted first? 1. Cervical dilation 2. Bladder distention 3. FHR pattern 4. Maternal BP

4. "As a nurse, I am legally bound to report abuse. I will stay with you while you give the report and help find a safe place for you to stay."

An 87-yr-old woman is brought to the ED for treatment from a fractured arm. On physical assessment, the nurse notes old and new ecchymotic areas on the client's chest and legs and asks the client how the bruises were sustained. The client, although reluctant, tells the nurse in confidence that her son frequently hits her if supper is not prepared on time when he arrives home from work. Which is the most appropriate nursing response? 1. "Oh, really. I will discuss this situation with your son." 2. "Let's talk about the ways you can manage your time to prevent this from happening." 3. "Do you have any friends that can help you out until you resolve these important issues with your son?" 4. "As a nurse, I am legally bound to report abuse. I will stay with you while you give the report and help find a safe place for you to stay."

3. Encourage the client to discuss the use of an herbal substance with the health care provider.

An antihypertensive medication has been prescribed for a client with hypertension. The client tells the clinic nurse that they would like to take an herbal substance to help lower their blood pressure. The nurse should take which action? 1. Tell the client that herbal substances are not safe and should never be used. 2. Teach the client how to take their blood pressure so that it can be monitored closely. 3. Encourage the client to discuss the use of an herbal substance with the health care provider. 4. Tell the client that if they take the herbal substance they will need to have their blood pressure checked frequently.

1. Naloxone 363 (410)

An opioid analgesic is administered to a client in labor. The nurse assigned to care for the client ensures that which med is readily available if respiratory depression occurs? 1. Naloxone 2. Morphine sulfate 3. Betamethasone 4. Mepreidine hydrochloride

1. Delivery of the fetus 308 (364)

An ultrasound is performed on a client at term gestation who is experiencing moderate vaginal bleeding. The results of the ultrasound indicate that abruptio placentae is present. On the basis of these finding, the nurse should prepare the client for which anticipated prescription? 1. Delivery of the fetus 2. Strict monitoring of I&O 3. Complete bed rest for the remainder of the pregnancy 4. The need for weekly monitoring of coagulation studies until the time of delivery

33 drops per minute 149 (193)

Gentamicin sulfate, 80mg in 100mL NS is to be administered over 30 minutes. The drop factor is 10 drops/mL. The nurse sets the flow rate at how many drops per minute?

4. Being affected by Rh incompatibility 364 (410)

Rh immune globulin is prescribed for a client after delivery and the nurse provides information to the client about he purpose of the med. The nurse determines that the woman understand the purpose if the woman states that it will protect her next baby from which condition? 1. Having Rh-positive blood 2. Developing a rubella infection 3. Developing physiological jaundice 4. Being affected by Rh incompatibility

1. We will be sure not to leave hot liquids unattended 215 (266)

The 2 year old child is treated in the ED for a burn to the chest and abdomen. The child sustained the burn by grabbing a cup of hot coffee that was left on the kitchen counter. The nurse reviews safety principles with the parents before discharge. Which statement by the parents indicates an understanding of measures to provide safety in the home? 1. We will be sure not to leave hot liquids unattended 2. I guess my children need to understand what the word hot means 3. We will be sure that the children stay in their rooms while we work in the kitchen 4. We will install a safety gate as soon as we get home so the children cannot get into the kitchen

2. Activate the emergency response plan 139 (184)

The Ed nurse receives a telephone call and is informed that a tornado has hit a local residential area and that numerous casualties have occurred. The victims will be brought to the ED. The nurse should take which initial action? 1. Prepare the triage rooms 2. Activate the emergency response plan 3. Obtain additional supplies from the central supply department 4. Obtain additional nursing staff to assist in treating the casualties

4. Initiate a gentle upward tap on the cervix 258 (310)

The HCP is assessing the client for the presence of ballottement. To make this determination, the HCP should take which action? 1. Auscultate for FHS 2. Assess the cervix for compressibility 3. Palpate the abdomen for fetal movement 4. Initiate a gentle upward tap on the cervix

1. Reflecting a cultural value

The ambulatory care nurse is discussing pro-op procedures with a Japanese American client who is scheduled for surgery the following week. During the discussion the client continually smiles and nods the head. How should the nurse interpret this nonverbal behavior? 1. Reflecting a cultural value 2. An acceptance of the treatment 3. Client agreement to the required procedures 4. Client understanding of the pro-op procedure

1. "Can you tell me what you understand about the procedure?"

The client is scheduled for an angioplasty. The client says to the nurse, "I'm so afraid that it will hurt and will make me worse off than I am." Which response by the nurse is therapeutic? 1. "Can you tell me what you understand about the procedure?" 2. "Your fears are a sign that you really should have this procedure." 3. "Those are very normal fears, but please be assured that everything will be okay." 4. "Try not to worry. This is a well-known and easy procedure for the health care provider."

4. Fluid overload 378 (426)

The clinic nurse instructs parents of a child with sickle cell anemia about the precipitating factors related to sickle cell crisis. Which,if identified by the parents as a precipitating factor, indicates the need for further instruction? 1. Stress 2. Trauma 3. Infection 4. Fluid overload

1. A client who has a history of IV drug use 287 (341)

The clinic nurse is performing a psychosocial assessment of a client who has been told that she is pregnant. Which assessment finding indicates to the nurse that the client is at risk for contracting HIV? 1. A client who has a history of IV drug use 2. A client who has a significant other who is heterosexual 3. A client who has a history of STI's 4. A client who has had one sexual partner for the past 10 years

4. Punishment and reward 201 (252)

The clinic nurse is preparing to discuss the concepts of Kohlberg's theory of moral development with a parent. What motivates good and bad actions for the child at preconventional level? 1. Peer pressure 2. Social pressure 3. Parents' behavior 4. Punishment and reward

4. I should avoid eating foods that produce gas and fatty foods 276 (324)

The clinic nurse is providing instructions to a pregnant client regarding measures that assist in alleviating heart burn. Which statement by the client indicates an understanding of the instruction? 1. I should avoid between meal snacks 2. I should like down for an hour after eating 3. I should use spices for cooking rather than using salt 4. I should avoid eating foods that produce gas and fatty foods

1. The child is 18 months old Children have to be older than 2 to use this medication 370 (420)

The clinic nurse is reviewing the HCP prescription for a child who has been diagnosed with scabies. Lindane has been prescribed for the child. The nurse questions the prescription if which is noted in the client's record? 1. The child is 18 months old 2. The child is being bottle-fed 3. A sibling is using lindane for the treatment of scabies 4. The child has a history of frequent respiratory infection

1,2,4 241 (294)

The clinic nurse prepares to perform a focused assessment on a client who is complaining of symptoms of a cold, a cough, and lung congestion. Which should the nurse include for this type of assessment? Select all that apply 1. Auscultating lung sounds 2. Obtaining the client's temperature 3. Assessing the strength of peripheral pulses 4. Obtaining information about the client's respirations 5. Performing a musculoskeletal and neurological examination 6. Asking the client about a family history of any illness or disease

2,3,6 133 (184)

The community health nurse is providing a teaching session about terrorism to members of the community and is discussing information regarding anthrax. The nurse tells those attending that anthrax can be transmitted by which routes? Select all that apply 1. Bites from ticks or deer flies 2. Inhalation of bacterial spores 3. Through a cut or abrasion in the skin 4. Direct contact with an infected individual 5. Sexual contact with an infected individual 6. Ingestion of contaminated under-cooked meat

1,2,4 283 (340)

The home care nurse is monitoring a pregnant client with gestational HTN who is at risk for preeclampsia. At each home care visit, the nurse assesses the client for which classic signs of preeclampsia? Select all that apply 1. Proteinuria 2. HTN 3. Low-grade fever 4. Generalized edema 5. Increased pulse 6. Incrased RR

1. Neglecting personal grooming 224 (274)

The home care nurse is visiting an older client whose spouse died 6 months ago. Which behavior by the client indicates ineffective coping? 1. Neglecting personal grooming 2. Looking at old snapshots of family 3. Participating in a senior citizens' program 4. Visiting their spouse's grave once a month

2. The child does not experience pain at the primary tumor site

The pediatric nurse specialist provides a teaching session to the nursing staff regarding osteosarcoma. Which statement by a member of the nursing staff indicates a need for information? 1. The femur is the most common site 2. The child does not experience pain at the primary tumor site 3. Limping,if a weight-baring limp is affected, is a clinical manifestation 4. The symptoms of this disease in the early stage are almost always attributed to normal growing pains

3. Enlarged, hardened veins 333 (380)

The postpartum nurse is assessing a client who delivered a health baby by cesarean for signs and symptoms of superficial venous thrombosis. Which sign would the nurse note if superficial venous thrombosis were present? 1. Paleness of the calf area 2. Coolness of the calf area 3. Enlarged, hardened veins 4. Palpable dorsalis pedis pulses

2. Continue to breast-feed every 2-4 hours 346 (398)

The postpartum nurse is providing instruction to the mother of a newborn with hyperbilirubinemia who is being breast-fed. The nurse should provide which most appropriate instruction to the mother? 1. Feed the newborn less frequently 2. Continue to breast-feed every 2-4 hours 3. Switch to bottle-feeding the infant for 2 weeks 4. Stop breast-feeding and switch to bottle-feeding permanently

1. 3 days 322 (373)

The postpartum nurse is providing instructions to a client after delivery of a healthy newborn. Which time frame should the nurse relay to the client regarding the return of bowel function? 1. 3 days 2. 7 days 3. On the day of delivery 4. Within 2 weeks

4. Increase hydration by encouraging oral fluids 320 (373)

The postpartum nurse is taking the VS of a client who delivered a healthy newborn 4 hours ago. The nurse notes that the client's temp is 100.2. What is the priority nursing action? 1. Document the finding 2. Retake the temp in 15 minutes 3. Notify the HCP 4. Increase hydration by encouraging oral fluids

4. Educating the client about therapies that the or she is using or is interested in using.

The role of the nurse regarding complementary and alternative medicine should include which action? 1. Advising the client about "good" versus "bad" therapies. 2. Recommending herbal remedies that the client should use. 3. Discouraging the client from using any alternative therapies. 4. Educating the client about therapies that the or she is using or is interested in using.

3. Lesions most often are located on the arms and chest Lesions are most often around the mouth and nose 369 (419)

The school nurse has provided an instructional session about impetigo to parents of the children attending school. Which statement, if made by a parent, indicates a need for further instruction? 1. It is extremely contagious 2. It is most common in humid weather 3. Lesions most often are located on the arms and chest 4. It might show up in an area of broken skin, such as an insect bite

4. White sacs attached to the hair shafts in the occipital area 372(420)

The school nurse is conducting pediculosis capitis (head lice) assessments. Which finding indicates a child has a "positive" head check? 1. Maculopapular lesions behind the ears 2. Lesions in the scalp that extend to the hairline or neck 3. Which flaky particles through the entire scalp region 4. White sacs attached to the hair shafts in the occipital area


Kaugnay na mga set ng pag-aaral

Chapter 66: Caring for Clients with Burns

View Set

CH04: Carbohydrates, HN 196 Chapter 4 - Carbohydrates, mastering nutrition 4-6, chapter 3

View Set

Fundamentals Test 2 Prep U Oxygenation

View Set

5 : Biochemical activity of the thyroid and parathyroid hormones

View Set

Handling and Restraint Self Check 1

View Set

NRRPT Prep Applied Radiation Protection

View Set